Sei sulla pagina 1di 108

The Osler Institute

GENERAL SURGERY REVIEW MOCK ORAL HANDOUTS


How to Pass the Oral Boards ................................... Joel Goldberg, MD Passing the Oral Surgical Board Exam ...........Robert Youngblood, MD Breast Topics for the Oral Boards...................... Charles Goldman, MD Sarcoma for the Oral Boards.............................. Charles Goldman, MD Melanoma for the Oral Boards........................... Charles Goldman, MD Adult Cardiac Surgery........................................Warren Widmann, MD Pediatric Cardiac Surgery...................................Warren Widmann, MD Pulmonary: Benign & Neoplasia ......................Warren Widmann, MD Vascular Surgery: Aortic....................................Warren Widmann, MD Vascular Surgery: Venous..................................Warren Widmann, MD Endocrine Oral Board Questions ....................... Charles Goldman, MD Esophageal Surgery............................................ Charles Goldman, MD Stomach for the Oral Boards.............................. Charles Goldman, MD Colon Surgery .................................................... Charles Goldman, MD Rectal Surgery .................................................... Charles Goldman, MD Gastrointestinal Bleeding ................................... Charles Goldman, MD Hepatobiliary Oral Board Questions.................. Charles Goldman, MD Pancreas Oral Board Questions.......................... Charles Goldman, MD Spleen ................................................................. Charles Goldman, MD Faculty Cases and Answers..................................................................... page 3 page 9 page 11 page 17 page 21 page 27 page 31 page 35 page 39 page 43 page 47 page 53 page 59 page 65 page 71 page 75 page 79 page 83 page 87 page 89

Surgery Mock Oral Handouts

Copyright 2010, The Osler Institute All rights reserved

Published by The Osler Institute 1400 E. Crossing Boulevard Terre Haute, Indiana 47802 www.osler.org/

Typeset by Readmore Bookstore 618 Wabash Ave Terre Haute, Indiana 47807 www.readmore.biz

Printed by Presstime Graphics, Inc. 1016 Poplar Street Terre Haute, Indiana 47807 www.presstime.com/

Without limiting the rights under copyright reserved above, no part of this publication may be reproduced, stored in or introduced into a retrieval system, transmitted in any form, or by any means (electronic, mechanical, photocopying, recording, or otherwise), without the prior written permission of both the copyright owner and the above publisher of this book.

The Osler Institute

How to Pass the Oral Boards


Joel Goldberg, MD Harvard University

I.

American Board of Surgery A. Written Boards (qualifying) - tests your knowledge. 1. Cant test judgment. B. Oral Exam (certifying) - tests your judgment and knowledge. 1. Are you rational? 2. Are you safe? 3. Are you ethical? 4. Do you have sound knowledge and decision making.

II.

The Examiners: there are two of them per room A. Permanent member of the ABS - - -chances are, if youve read the textbooks you will know this guy! 1. Hes usually very nice - he knows he knows more than you. 2. Hes also very smart - dont BS him. B. Associate Examiner (local guy) - no clue who this turkey is! 1. Hes usually not as nice as the first guy. 2. Trying to prove to the first guy that he knows more than you. 3. Doesnt always know more than you. Nonetheless dont BS or challenge him either.

III.

The Topics most frequently addressed: three rooms along these lines. A. Upper Gastrointestinal/Endocrine/Soft Tissue room 1. Esophagus: Boerhaaves, achalasia, Zenkers, Barretts, cancer 2. Stomach: afferent loop syndrome, non-healing gastric ulcer, gastric outlet obstruction 3. Pancreas: painless jaundice, pancreatic abscess, pseudocyst, splenic vein thrombosis, Infected Pancreas, Necrosis/acute pancreatitis, 4. Duodenum: perforated, bleeding DU 5. Liver and Biliary tree: cholangitis, CBD injury, solitary liver mass 6. Spleen: ITP, post splenectomy abscess with pancreatic fistula

GS_SDS_Oralrevised03012010 (6/2/2010)

GS-SDS-Oral 2010

page 3

Surgery Review Course


7. Melanoma: extremities vs. trunk, SLNB vs. node dissection vs. mets 8. Sarcoma: extemities vs. retroperitoneal, XRT/chemo 9. Endocrine: thyroid nodule, primary HPTH, Pheochromocytoma, MEN syndromes, Gastrinoma B. Trauma and Critical Care room 1. Burns 2. ARDS 3. Pelvic Fracture 4. Pulmonary Embolus 5. Trans mediastinal GSW 6. Stab wound neck 7. ACLS/ATLS 8. Post-op MI 9. Necrotizing Fascitis 10. Abdominal Compartment 11. Oliguria p-op AAA: Dx is ischemic colitis/cold leg/ARF/MI/CHF/ARDS/Pneumonia/acute ccy/ acute pancreatitis as all of these can present with low u/o, acidosis, hypoxia and tender abdomen. C. Lower GI/Vascular/Breast/Endocrine/Pediatric room 1. Small Intestine: Crohns disease, enterocutaneous fistula, SBO, early prop SBO 2. Colon: ischemic colitis, UC, Toxic Megacolon, colon cancer at AAA, diverticulitis, colovesical fistula, LGIB 3. Appendix: octopus, i.e. every variation of RLQ pain 4. Rectum: low rectal CA, perianal DZ, anal cancer. 5. Vascular a. Cold leg b. Ruptured AAA c. Renovascular Hypertension d. Mesenteric ischemia; art vs. ven; acute vs. chronic e. Carotid artery disease f. Diagnosis and work up of DVT 6. Breast: Everything. Everyone gets a breast question. 7. Pediatrics: testicular torsion vs. hernia. Traumatic duodenal hematoma. 8. Thoracic: solitary pulmonary nodule, empyema, thymoma
page 4 GS-SDS-Oral 2010
GS_SDS_Oralrevised03012010 (6/2/2010)

The Osler Institute


D. Oddball Questions: pelvic mass at splenectomy, colon CA at AAA, Neurosurgical patient with PE, GI bleed with PE, Pregnant woman with Breast Cancer, No u/o post-op, TAH-BSO IV. Conduct of the room A. Examiners will alternate questions generally wont tag team you on a question B. Briefly answer each point they ask, and wait for their response, and then describe your next step. C. Its a tennis game, not a chess game or duck hunting w/shotgun. D. Dont ramble on about all possible outcomes or scenarios, state what you would do and wait for their response. Let them conduct the course of the question. E. Rarely, multiple equal answers exist, and you need patient input. You should then offer the options and ask for patient input, e.g. Breast CAI would discuss with the patient the options of MRM vs. lump ax with XRT, if the pt were a good candidate. F. They want to cover 4 questions G. If you finish early: dont worry, if you get to five questions: dont worry. V. Scoring the room A. Scale 7 exceptional 6 definitely pass 5 maybe pass; can move up 4 maybe fail; can move down 3 definitely fail B. All six examiners will discuss you C. You need to pass 2/3 rooms D. Dont need to pass the trauma critical care room E. You will not have any of the following or anyone you know personally as an examiner. Chairman, Program director, Lab Director, Faculty

GS_SDS_Oralrevised03012010 (6/2/2010)

GS-SDS-Oral 2010

page 5

Surgery Review Course


VI. Day of the Exam A. Dress Conservatively: no mini skirts, outlandish ties etc. B. Be on Time: if you are not staying at the hotel where the exam is, make sure you go to the hotel the night before and know how to get there. C. Dont knock on the door: they will come and get you (they are probably still devouring the last guy like a pack of rabid dogs). D. Shake hands firmly, be friendly. E. Sit up, dont slouch. F. Dont fidget G. Make eye contact. H. Dont confront them or challenge I. When you dont know: SAY YOU DONT KNOW, NEVER BS THEM. J. ALWAYS BE ETHICAL: you could nail every ? but if you lie to a patient, you FAIL. K. Thank them at the end. VII. Answering the Questions A. Listen carefully to the question because they will usually give you prompters that lead you to the diagnosis: they want to see how you manage the problem. B. Ask any pertinent additional history - -positives and negatives. If they tell you that you dont need any more H & P, then you dont. C. Ask for labs and X rays D. Ask for appropriate tests to rule out or rule in your concernsif you dont get a definitive answer, maybe you need a more definitive test. Maybe youll end up with an ex-lap if thats the safe thing. E. Cancer patients 1. Local disease 2. Regional Disease 3. metastatic work upboth pre-op and intra-op. Remember to re-stage them if they develop a recurrence, e.g. dont go for the APR with a
page 6 GS-SDS-Oral 2010
GS_SDS_Oralrevised03012010 (6/2/2010)

The Osler Institute


small recurrent anal squamous cell after Nigro protocol until you rule out distant mets. 4. Chemotherapy for common cancers, e.g. breast, colon. F. trauma patients: ATLS always, ABCs. G. Always go and examine the patient. H. If non-operative management is the preferential treatment thats fine. In fact, in ICU rooms you may never get to the OR. However, they will probably push you to operate; after all it is the ABS exam. I. You might kill the patient: thats OK. Some stems end that way and they want to see how you respond under pressure. J. Never make up an operation: If youve never done that operation, or if it out of your scope of practice, then say so, but give them the general principles of the operation. K. Know how to manage complications. (Relax, they are not your fault - most questions will include a possible complication.) L. OK to go back, just not back to the last question/previous room. M. Operations youll need to know how to do. If you say, I dont know to these, youre in trouble. 1. Whipple 2. Esophagectomy 3. Right Hepatic Lobectomy 4. Thyroid nodule 5. Trauma lap: exposure Liver, Aorta, visceral vessels, RP structures 6. Neck exploration. 7. IPAA 8. APR 9. Surgical management of perforated Esophagus a. Cancer b. Achalasia c. Boerhaaves VIII. How to fail the exam Complete disorganization and inability to get through enough of the question to demonstrate your knowledge.

GS_SDS_Oralrevised03012010 (6/2/2010)

GS-SDS-Oral 2010

page 7

Surgery Review Course


Not knowing how to describe one of the big cases mentioned above. Not assessing ABCs or resuscitating the patient appropriately before diving into the problem. Jumping right to the OR without evaluating the patient or non-operatively managing when its indicated. (Although this is the ABS exam, how and when to operate is more important that regurgitating the name of the procedure). Arguing with the examiners, being cocky, or being otherwise unpleasant. Performing unnecessary or contraindicated procedures. e.g. Whipple or pelvic exenteration without ruling out distant mets both pre-op and intraop. Resecting an esophageal CA invading the trachea. Early re-op for gastroparesis from gastrojejunostomy) Not doing the safe thing. (If your patient is having a complication postoperatively, dont try to impress them on the art of risk-taking and observation when something definitive should be done.)

Final Thoughts
NO TRICKS QUESTIONS. DONT MAKE UP STUFF. IF YOU DONT KNOW THEN SAY SO. REMEMBER ACLS/ATLS BE SAFE

page 8

GS-SDS-Oral 2010

GS_SDS_Oralrevised03012010 (6/2/2010)

The Osler Institute

Passing the Oral Surgical Board Exam


Robert Youngblood, MD East Carolina University

I. PASSING The Oral Surgical Board Exam


Your objective in taking a preparatory course is to pass the General Surgery Oral Board Exam. Your examiners purpose is to access your clinical judgement and the ability to apply cognitive knowledge to practical surgical problems. My purpose is to do all that I can to assist you in passing. It is important for you to outline your thought processes in arriving at a diagnosis and describing your management decisions.

A. The American Board of Surgery expects you to be knowledgeable and proficient in five areas:
You must have a central core of knowledge in Anatomy, Physiology, Pathology, Immunology, Nutrition, Metabolism, Shock Resuscitation, Intensive Care and Wound Healing. You must possess the comprehensive knowledge and skill in the diagnosis and pre-intra and postoperative care in 9 areas. (Alimentary tract, abdominal contents, skin and soft tissue including breast, head, and neck, vascular system, endocrine system, surgical oncology, trauma management and critical care.) You should be significantly familiar with the surgical management of pediatric, general thoracic, transplant and plastic surgical patients. You must understand the management of the more common problems in anesthesia, cardiac surgery, gynecology and neuro, orthopedic and urologic surgery. You must be able to perform endoscopic techniques including bronchoscopy, laryngoscopy, upper gastrointestinal endoscopy, colonoscopy and basic laparoscopic surgery.

B. The examinations are conducted by two examiners, both of which are board-certified surgeons, and one of whom is a member of the American Board of Surgery.
1. A few practical points: I would suggest your initial impression is important. Dress conservatively; shake hands firmly; establish eye contact; be friendly, smile conservatively and treat your examiner with respect. You will be presented a series of cases to discuss. Pause briefly and organize your thoughts. Proceed in logical
GS_SDS_Oralrevised03012010 (6/2/2010)

GS-SDS-Oral 2010

page 9

Surgery Review Course


order evaluating history, physical, laboratory studies and special studies. Establish in your mind a diagnostic decision tree and outline for your examiner the purpose, result, and significance of the test you do in establishing the diagnosis. Likewise, describe in logical order the therapeutic steps eliciting how and why you do what you do. Do not omit preoperative preparations if indicated such as cardiac evaluation, pulmonary toilet electrolytes, blood typing, antibiotics, nutritional evaluations, etc. Your examiner may proceed with a line of questioning until he stumps you. Be careful do not B.S.! Tell him how you would go about finding the information you are uncertain about or seek consultation with appropriate authorities. If you are quite knowledgeable and very comfortable with a clinical problem your examiner may stop you and go to a different subject. You may be wise to deal thoroughly and deliberately with the situation in which you are knowledgeable but do not give the appearance of stalling. Listen to the facts of the case carefully as presented and be alert for prompters both in the original presentation and in subsequent questions. Prompters are subtle clues be alert for them. 2. The following is a list of things the examiner is looking for: a. Do you recognize the basic problem? b. Do you analyze and interpret the data correctly? c. Do you ask clarifying questions and seek appropriate additional data? d. How is your problem solving ability? e. What is your basic knowledge level? f. Do you choose realistic, effective therapies? g. Do you anticipate and manage complications? h. Do you adopt alternative therapies, if needed? i. Do you communicate effectively? j. Are you a safe surgeon? k. Do you know when not to operate? 3. To pass the examination, you must have basic knowledge; but how you organize and apply that knowledge is equally important. In summary, appear respectful, friendly and confident. Address the question you are asked. Organize your replies and verbalize your thought processes. Outline not only what and how, but why you do what you do. Be alert for prompters. Good luck, but good preparation is better.

page 10

GS-SDS-Oral 2010

GS_SDS_Oralrevised03012010 (6/2/2010)

The Osler Institute

Breast Topics For the Oral Boards


Charles D. Goldman, MD Des Moines University/Mercy Medical Center Des Moines

Initial interview of patient: Get family history of malignancy, especially early onset breast (including male relatives) and ovarian at any age. Softer link to early onset colon and prostate. All are associated with BRCA tumor suppressor gene deletions. Look for pattern of multiple firstdegree family members with these associated malignancies and at least one with early age of onset (prior to age 50). Early onset ovarian cancer has especially ominous prognosis. Ask about age of first full-term pregnancy (earlier the better) and age of menarche (later the better). Nulliparity or first pregnancy after age 35 increases risk of breast cancer. Work-up: Make diagnosis of cancer before staging for distant disease. There should be almost no employment of open biopsy, except for those who cannot undergo stereotactic biopsy. Core biopsy now standard, FNA rarely used. Advantage of core needle biopsy is ability to display invasion (and thereby R/O DCIS) and to obtain receptor analysis, including her-2 neu status. Mammography is not a diagnostic test; to call mass benign or malignant, look for triple concordance of tissue diagnosis, physical exam and mammography. Ultrasound is an adjunctive test. If mass palpable but negative mammography, get biopsy unless has all characteristics of simple cysts or fibroadenoma on ultrasound. DCIS: Characterized as high or low grade depending on nuclear changes and presence/absence of comedonecrosis. Not clear that any more than 25% of these lesions will ever progress to frank carcinoma; therefore, many observers feel less aggressive management, especially omission of radiotherapy, in the older patient can be justified. Although standard is to offer radiotherapy to all DCIS patients, no evidence of any survival benefit to adding radiation in DCIS. Aim for clear margin excision; if disease is multifocal this may be impossible. Recent NCCN guidelines state that while 1 cm margin is optimal, clear margin is adequate (<1 mm) in constrained sites such as against chest wall or at skin. Although controversial, radiation therapy is generally added to excision except for small low grade DCIS excised with at least 1 cm margin in the older patient. Partial breast irradiation has been used on protocol. Tamoxifen reduces
GS_SDS_Oralrevised03012010 (6/2/2010)

GS-SDS-Oral 2010

page 11

Surgery Review Course


both invasive and DCIS recurrence, but has no effect on survival. It should be considered in all patients with highgrade ER (+) DCIS unless there is a contraindication such as concern for endometrial cancer or prior thromboembolic disease. Mastectomy not needed except for extensive DCIS that cannot be encompassed by lesser surgery, up to and including quadrantectomy. Lymph node surgery generally not needed in DCIS. SNB in DCIS is not standard of care except in setting of microinvasion, mass-associated DCIS, or when doing mastectomy for DCIS (ASCO and NCCN guidelines). LCIS: Lobular carcinoma in situ is marker for enhanced lifetime risk of developing infiltrating cancer, usually of ductal variety. Risk is same in both breasts and is not associated with the location that the LCIS is encountered. Generally LCIS is an incidental finding on biopsy done for other reasons; not associated with microcalcifications. Unless patient is difficult to follow closely or has LCIS along with significant family history, intensive surveillance is best option, perhaps adding MRI if patient is difficult to assess with mammography. Consider chemoprophylaxis with tamoxifen for about 50-60% proportional risk reduction. In very high risk patients, bilateral simple mastectomies will reduce risk of subsequent cancer about 95%. Staging: Stage Iprimary smaller than 2 cm, no nodes; Stage II A-regional nodal disease or tumor from 2-5 cm with no nodes; Stage II Bprimary from 2-5 cm with nodes, or >5 cm tumor no nodes; Stage III A- tumor > 5 cm with nodes, or tumors with bulky but operable axillary adenopathy; Stage IIIBsigns of inoperability such as chest wall invasion, peau dorange, ulceration and inflammatory breast cancer; supraclavicular adenopathy; Stage IV-metastatic disease Early breast cancers (Stage I-IIA): Stage prior to surgery with serum liver function tests and chest X-ray. No need for bone scans or CTs unless patient has suggestive symptoms or lab abnormality. All lesions < 5 cm can be considered for conservative therapy, even if nodes clinically positive. May even consider larger lesion for conservation if large breast size would allow for reasonable cosmetic results and no signs of unresectability. Send to radiotherapist preoperatively to assess for any contraindication to radiotherapy, e.g. multiple quadrant disease, high tumor to breast size ratio, collagen vascular disease. Remember that for lesions < 5 cm mastectomy and lumpectomy with radiation are equivalent therapies; always lean towards breast conservation where appropriate. Sentinel node biopsy is now standard of care for
page 12 GS-SDS-Oral 2010
GS_SDS_Oralrevised03012010 (6/2/2010)

The Osler Institute


evaluation of regional disease in these patients. Combination of blue dye and tracer gives highest yield, although additional yield of blue dye wanes with experience. Board answer is to complete node dissection on all those patients found to have macrometastases (>2 mm) even if only one node positive. Axilla management in setting of micro mets (.2-2mm) or nano metastases (isolated tumor cells or <.2 mm) unclear. For Board, would likely complete node dissection for micromets but not for nanomets . Adjuvant therapy recommendations vary with the week but the general tendency is not to offer node-negative patients with any tumor < 0.5 cm or a well-differentiated tumor < 1 cm adjuvant therapy. In those node negative patients with ER (+) tumors, Oncotype testing (gene expression microarray) may be used to generate a risk score and, if high, chemotherapy would be offered. Similarly, if these tumors are unfavorable (steroid receptor negative, have lymphovascular invasion or express her-2 neu oncogene), many oncologists would offer chemotherapy followed by 5 years of tamoxifen to all premenopausal patients and an aromatase inhibitor (Femara, Arimidex) in the postmenopausal patient. Most common chemotherapy regimen is Adriamycin Cytoxan (AC) with decreasing use of cytoxan-methotrexate-5-FU (CMF). All Stage II patients get an additional 4 cycles of a taxane, Taxotere in most cases. Value of chemotherapy never been validated in elderly due to limited participation in clinical trials. In high risk early breast cancers (poorly differentiated, multiple nodes involved, younger patient, etc.), Herceptin with a taxane and platinum agent (TCH) or AC followed by TH (taxotere/Herceptin) is given as adjuvant therapy instead of AC-T in patients who are her-2 positive. Treatment of the triple negative tumors with chemotherapy much less successful. In patients with greater than 3 nodes positive and/or primary > 4 cm in size, chest wall, axillary and supraclavicular radiation +/- internal mammary radiation should be offered even if patient has undergone mastectomy, unless there is a contraindication to radiation such as significant cardiopulmonary disease or prior radiation. Locally advanced breast cancer (Some Stage IIBs and Stage III): Standard treatment is preoperative chemotherapy in hopes of shrinking the primary tumor to allow more conservative breast surgery. Some evidence of survival benefit vs. postoperative therapy even if not converted to more conservative surgery. Assess patient prior to chemotherapy for distant disease (CT/PET is not routinely recommended per NCCN), ultrasound axilla and biopsy any suspicious nodes, FNA any palpable suspicious nodes, get MRI to better assess extent of tumor/monitor response
GS_SDS_Oralrevised03012010 (6/2/2010)

GS-SDS-Oral 2010

page 13

Surgery Review Course


especially if lobular pathology. Most breast surgeons are now doing sentinel node after chemo for those who present with a clinically negative axilla or convert to one. Chemotherapy will usually involve use of at least three agents, Adriamycin, Cytoxan and a taxane, either in various combinations or sequentially. If her-2 positive, alternatively TCH or AC-TH will be offered. If shrink to small enough volume, can consider breast conservation. Otherwise offer mastectomy. No survival benefit to mastectomy vs appropriately margined local excision. Patients should receive full axillary dissection if nodes were clinically positive on presentation and remain so during treatment or turn positive during treatment. Depending on extent of residual tumor encountered, more chemotherapy may be offered, although benefit not determined. Persistence of disease in axillary nodes is more ominous than persistence of primary mass. All patients, even those who have a mastectomy, are given comprehensive chest wall and nodal radiotherapy. Place on tamoxifen or aromatase inhibitor if receptor positive tumor. The approach is similar for inflammatory breast cancer, although in this disease some may offer whole breast radiation in lieu of modified radical mastectomy after initial very intensive chemotherapy. Some evidence that mastectomy followed by chest wall RT gives superior results to RT only in inflammatory cancer. Immediate reconstruction can be offered to those patients who appear to have a complete response after neoadjuvant chemo, but not to those with obvious persistent tumor as their local recurrence rates exceed 50%. How to reconstruct is a major question due to need for chest wall radiation. Pregnancy-associated breast cancer: Actually not a difficult question. Key points are that chemotherapy can be given during pregnancy after the late first trimester (generally avoiding the antimetabolites such as methotrexate, taxanes, and Herceptin) and terminating the pregnancy offers no survival advantage. Although chest wall or breast radiation therapy is not considered safe during any trimester, since these patients almost always have Stage II or greater disease they will need to undergo up to 24 weeks of chemotherapy before they are ready for radiation. Therefore, conservation therapy can be offered to appropriate patients even if they present during pregnancy since the radiation is delayed until after delivery by the use of chemotherapy. Tamoxifen cannot be given during pregnancy. Mammography and chest X-ray are safe to use, CT is relatively contraindicated (use MRI without gadolinium in preference to CT) and nuclear med scans such as a bone scan are absolutely contraindicated. Radiotracer for sentinel node biopsy is safe (the radiation
page 14 GS-SDS-Oral 2010
GS_SDS_Oralrevised03012010 (6/2/2010)

The Osler Institute


dose is infinitesimal) but blue dye is not. NCCN guidelines still recommend axillary dissection in preference to SNB, but the latter is being widely used in pregnancy already. Stage IV breast cancer: No evidence that aggressive therapy, such as high-dose chemotherapy with bone marrow/stem cell support, is any better than more standard measures. If patient asymptomatic, discovery of metastases on a scan does not necessarily justify treatment. In receptor positive tumors, salvage therapy is with second-line hormonal agents (assuming patient has failed on Tamoxifen) such as aromatase inhibitors, LHRH agonists that induce medical ovarian ablation (Zoladex) in the premenopausal patient, Faslodex (an irreversible inhibitor of the estrogen receptor) or testosterone/estrogen/Megace. Recent studies would seem to favor aromatase inhibitors as first choice. Symptomatic bony mets are generally radiated and Aredia/Zometa (bisphosphonates that serve as osteoclast poison) is added to reduce bone pain and may also have tumoricidal activity. For symptomatic visceral metastases (esp. lung and liver), further chemotherapy might be useful. If patients tumor expresses her-2/neu oncogene add Herceptin to chemotherapy. EGF and VEGF inhibitors also starting to see use in metastatic breast cancer. Essentially all these patients will die of disease within 2-5 years, unless they can be treated with Herceptin, have bone only disease, or have endocrine responsive tumors. Although there is anecdotal evidence for, and ongoing research on, metastectomy, for the Board would likely not offer lung or liver metastasectomy for seemingly isolated mets. Recent literature appears to show survival benefit to revmoval of the primary tumor even in the presence of Stage IV disease. Pagets disease: Eczematous changes that are bilateral and/or do not involve the nipple are likely to be benign (Pagets almost always involves the nipple proper). If there is any question of whether patient has Pagets, better to biopsy prior to prescribing topical steroids as Pagets can remit on steroids. Paget cells are felt to represent migration of malignant cells from underlying DCIS or invasive cancer rather than in situ transformation. Workup for occult mass with physical examination, mammogram and ultrasound. If no mass found, obtain an MRI. If all work-up is negative, options are simple mastectomy or NAC excision with wholebreast radiation. Axillary staging only if do mastectomy. If work-up reveals DCIS, simple mastectomy with axillary staging optional vs. DCIS excision, NAC excision and WBRT. Invasive tumor presentation gets same treatment as DCIS except axillary staging is mandatory.
GS_SDS_Oralrevised03012010 (6/2/2010)

GS-SDS-Oral 2010

page 15

Surgery Review Course


Breast cancer presenting as axillary node: FNA early to direct investigation. If adenocarcinoma, likely sources are breast, thyroid and lung. Remove node before doing multiple scans looking for occult primary. Immunohistochemistry on node (estrogen receptor, thyroglobulin, S-100, cytokeratin panel) should enable definitive determination of source organ of malignancy. If breast is seemingly primary, do usual staging work-up and get mammogram/ultrasound/MRI of breast. (If nodal pathology suggests abdominal source, only test of value is serum CA-125 in female to R/O ovarian primary. Chemotherapy for Stage IV ovarian will significantly prolong life. Waste of time and money looking for occult GI malignancy since this is already Stage IV disease and removing an asymptomatic primary will not prolong the patients life). If mammogram shows mass, can still offer breast conservation with lumpectomy, node dissection and radiation. Offer adjuvant chemo or hormonal therapy since patient is by definition Stage II disease. If no mass on breast imaging, U.S. surgeons generally do modified radical mastectomy, while Europeans tend toward axillary node dissection and whole breast irradiation. Once again, whatever local measures are used, adjuvant therapy is given for Stage II disease. Prognosis of patient presenting with axillary mass and no discernable primary is actually better than that for Stage II patients with localizable primary tumor. Local recurrence: In all cases, first step is to do staging work-up, usually involving bone scan and CTs of chest/abdomen. If parenchymal recurrence in radiated breast, do simple mastectomy. If skin recurrence in radiated breast (a rare occurrence but one that has abysmal prognosis), unclear whether local excision or simple mastectomy is better; assess whether patient can tolerate any further radiation to the chest wall. If skin recurrence after mastectomy, locally excise and radiate. Use of systemic therapy in this situation remains controversial. If already on tamoxifen, with hormone receptor positive tumor, might consider switching to second-line hormonal agents as noted above. Utility of chemotherapy in this scenario is uncertainsome oncologists would treat (? with what and for how long) even if local recurrence is fully excised and there is no evidence of metastatic disease, while others cite data showing no benefit for pre-emptive use of chemotherapy in the absence of non-regional disease.

page 16

GS-SDS-Oral 2010

GS_SDS_Oralrevised03012010 (6/2/2010)

The Osler Institute

Sarcoma For the Oral Boards


Charles D. Goldman, MD Des Moines University/Mercy Medical Center Des Moines

Key Points:
1. Prognosis is function of: size (>5 cm associated with worse outcome), grade (this is a subjective light microscopic evaluation based on nuclear atypia, presence/absence of necrosis and number of mitoses per 50 high power fields), and location of primary (retroperitoneal tumors worse prognosis not due to tumor biology but to limitations on achieving wide margined surgical excision). Of all factors, number of mitoses per 50 high power fields is probably most critical variable. If number of mitoses > 10, the lesion is a high-grade sarcoma. 2. Pathologic subtype of sarcoma (e.g., liposarcoma vs. malignant fibrous histiocytoma) is generally of little importance in either management or prognosis. Only
exception is the small-cell sarcomas (Ewings, primitive neuroectodermal tumors, rhabdomyosarcoma) whose relatively high sensitivity to chemo and radiotherapy can alter management scheme.

3. Lymph node involvement in sarcoma is rare, with clear cell, synovial, epithelioid, and the angiosarcomas having the highest rates. Even so, lymph node resections should only be done in the presence of gross involvement, never prophylactically. 4. High grade lesions tend to distantly metastasize (with lung as the most frequent initial site), often without evidence of recurrence of the primary, while the main risk in low grade sarcomas is local recurrence. 5. There has been general abandonment of amputational procedures for adult extremity sarcoma, except where this is made necessary by involvement of the hip, knee, elbow, or shoulder joints, or the bony pelvis. More aggressive surgery does not result in increased survival, simply better local control. Since the latter does not correlate with improved survival in high-grade sarcomas, the trend is toward conservative surgery with use of radiation therapy to augment surgery.

GS_SDS_Oralrevised03012010 (6/2/2010)

GS-SDS-Oral 2010

page 17

Surgery Review Course


6. Treatment schema developed for pediatric tumors (such as limb
perfusion) have generally not been useful in adult soft tissue tumors. In adult tumors, surgery is usually offered first followed by for use of postoperative radiotherapy, especially if sarcomas high-grade or there are positive margins. Almost all high-grade sarcoma patients will receive adjuvant radiation while only low grade sarcomas >5 cm in diameter will do so. Most patients with dermatofibrosarcoma protuberans will receive adjuvant RT even if margin negative. In either case, radiation only increases local control; there is no improvement in cancer-related mortality. In the case of the above mentioned small cell sarcomas or other high-grade sarcomas, can consider use of neoadjuvant chemoradiation to cytoreduce tumors especially if this will allow for preservation of limbs or vital neurovascular structures (e.g., sciatic nerve).

7. Chemotherapy for sarcomas remains highly controversial, and


it is likely of marginal benefit in most cases. Most active agents are epirubicin (an anthracycline related to Adriamycin) and ifosfamide (an alkylating agent similar to Cytoxan). Most recent statement regarding chemotherapy in sarcoma from American Society of Clinical Oncology (2000 Education Book): Adjuvant chemo (for sarcoma) should only be considered for Stage III lesions (high-grade pathology, T>5 cm, deep to superficial fascia) and it should not be used in terms of the board examination until the controversies are laid to rest.(But of course cannot guarantee it is not still the Surgery Board answer)

WORK-UP OF SUSPECTED SARCOMA


1. In extremity, get tissue prior to excision. FNA may be used but is generally better for assessing recurrence, rather than making initial diagnosis. Core needle biopsy can be employedbe sure to tattoo skin site where needle is placed so it can be excised with the sarcoma. Open biopsy is most often used. Do not shell out any lesion suspected to be sarcoma due to pseudo encapsulation. Can do excisional biopsy with 2 cm margin for lesions < 2-3 cm, for larger ones do incisional biopsy. Make biopsy incision parallel to muscle group so can be re-excised when resection done. Assure hemostasis to avoid tracking of tumor cells away from biopsy area and to avoid confounding MRI/CT visualization. 2. With retroperitoneal tumor (almost always diagnosed by CT or MRI obtained for non-specific complaints of abdominal or back pain) do not percutaneously biopsy unless there is a high likelihood this is something else other than sarcoma (e.g., a large retroperitoneal mass
page 18 GS-SDS-Oral 2010
GS_SDS_Oralrevised03012010 (6/2/2010)

The Osler Institute


associated with extensive peri-aortic adenopathy is far more likely to be lymphoma than sarcoma; in male check serum markers to rule out germ cell tumors and also consider pheochromocytoma) and making that diagnosis would obviate the need for surgical exploration 3. Staging work-up consisting of chest X-ray is probably adequate, but most physicians probably would get CBC and liver panel despite low rate of metastases to liver or bone. No value to getting bone scan or chest CT unless the less expensive baseline testing suggests possibility of involvement in those areas. 4. Physical examination (be careful to assess for any circulatory or neurologic deficit in the extremities, including presence of edema) and CAT scan is more than adequate work-up for an extremity sarcoma. MRI adds little that is not apparent from these two. In the retroperitoneum, MRI is more useful as it allows evaluation of the inferior vena cava that may be involved in bulky sarcomas (although multiarray CT reconstructions are probably just as good these days). In neither site is it mandatory to get a MRI if you already have a good quality CT.

THERAPEUTIC PRINCIPLES:
1. Goal of sarcoma surgery is a grossly clear margin. Given the often-immense size of these tumors, frozen section to check margins is worthless. The oft-quoted but never validated 2 cm margin is more easily achieved in the extremity, whereas in the retroperitoneum gross excision is about all that can be achieved. 2. Entire muscle group does not need to be removed if a clear margin can be gotten without resorting to that 3. In retroperitoneal sarcoma, do not remove organs/structures that are contiguous to the sarcoma in an attempt to get 2 cm margins. If the sarcoma is adherent to a structure, carefully dissect it free; on the other hand, if there is invasion by the sarcoma, resect en bloc. 4. Circumferentially mark the excision site with clips as external beam radiotherapy is often given as adjuvant treatment even if tumor completely excised. Presently, there is almost no indication for use of adjuvant RT in sizeable retroperitoneal sarcomas as the dose needed would be highly toxic to small bowel, kidney and liver.
GS_SDS_Oralrevised03012010 (6/2/2010)

GS-SDS-Oral 2010

page 19

Surgery Review Course


5. No proof that brachytherapy or intraoperative radiation therapy is superior to external beam RT. However, the former modalities may be useful when the area involved with residual disease is limited as you can then treat a small area without having to radiate the entire tumor bed. 6. Radiation can clean up microscopic residual disease, so reasonable to leave microscopic diseases if this allows you to save vital neurovascular structures. Extremity arteries are generally expendable as can be replaced with vein or conduit; femoral nerve can be sacrificed with minimal disability resulting, but sciatic nerve leaves patient with almost useless leg except in the young patient with good rehabilitation potential. Removing large central extremity veins often leaves patient with disabling limb edema. Can remove IVC if needed; oftentimes, if IVC was obstructed by tumor, patient has sufficient collaterals around it to allow for simple excision with no replacement. Otherwise replace with harvested internal jugular vein. Can also use Gore-Tex/Dacron graft and make temporary distal A-V fistula (usually saphenous end to side to femoral artery) to increase patency. 7. Approximately 60% of patients with recurrent sarcoma can be salvaged so treatment of all but the most widespread local or distant recurrences is re-resection. Pulmonary metastasectomy for sarcoma is an extensively studied model and is justifiable if primary disease site is controlled and number of pulmonary lesions is modest (no absolute maximum for # of mets that can be resected but most series stop at 5-8). Whether same principles apply to resection of the far less common liver mets is uncertain. 8. In a generally moribund field, biggest news is use of Gleevec, an oral small molecule tyrosine kinase inhibitor that has activity in sarcomas that express either the c-kit oncogene or PDGF. These constitute almost exclusively the gastrointestinal stromal tumors. Stomach is the most common location for these increasingly commonly seen neoplasms. Treatment is careful resection to avoid rupture of these fragile tumors and generally adjuvant Gleevec for high grade and larger tumors after testing for c-kit oncogene amplification.

page 20

GS-SDS-Oral 2010

GS_SDS_Oralrevised03012010 (6/2/2010)

The Osler Institute

Melanoma For the Oral Boards


Charles D. Goldman, MD DesMoines University/Mercy Medical Center Des Moines

Key Points:
1. Five main pathologic subtypes in order of worsening prognosis: lentigo maligna (Hutchinsons freckle), superficial spreading, acral lentiginous, nodular. Subtypes with long radial growth phases (lentigo, superficial spreading) have better prognosis, while those that enter vertical growth phase early in their natural history (nodular) do poorly. Desmoplastic melanomas, predominantly found on the head and neck, have prognosis intermediate between nodular and superficial spreading. 2. Breslow depth is most important factor in predicting prognosis; Clarks levels vary too much with differing skin thicknesses to be consistently useful. Other pathologic factors that bode poorly in decreasing order of importance: ulceration, high mitotic rate, presence of regression, absence of tumor invading lymphocytes. Male sex, older age and proximal location of the melanoma (trunk vs. extremity) portend a poorer prognosis. BANS area lesions are notoriously bad actors especially scalp and ear lesions. 3. Use TNM staging system: Stage I-primary < 2 mm depth, no nodes; Stage II-primary >2 mm, no nodes; Stage IIIregional nodal disease; Stage IV- distant metastases. New staging system divides T stages into T1=< 1 mm, T2 = 12 mm, T3 = 2-4 mm, T4 > 4mm. There is an a. and b. designation representing the absence/presence of ulceration or, in the case of T1 lesions only, Clarks level IV or greater. The nodal status is designated a. and b., differentiating whether the nodes are positive with micro or macro-disease. M status is subdivided into distant skin or nodal mets, lung mets, and all other visceral mets and/or LDH elevation.

Therapeutic Principles:
1. Do not shave biopsy suspected melanoma. If lesion small and not in cosmetically sensitive place, excise with 1-2 mm margin to avoid disruption of lymphatic drainage which might interfere with ability to do sentinel node
GS_SDS_Oralrevised03012010 (6/2/2010)

GS-SDS-Oral 2010

page 21

Surgery Review Course


biopsy later. If larger, punch biopsy, into the subcutaneous fat, the darkest or most heaped up portion of the lesion (basically guessing at where it is thickest). There are laser implements to actually scan melanomas for relative thickness but are not widely available outside research institutions. 2. If biopsy positive, do routine cancer staging. Examine all lymph node basins. Get chest X-ray and liver panel. Always include serum LDH in evaluation as this can be used to monitor for occult recurrence/metastases (akin to its value in non-Hodgkins lymphoma; reason unknown) and elevated LDH also serves to upstage disease in new AJCC staging system. If lymph nodes are clinically suspicious, FNA to confirm their involvement and then CT scan to evaluate next echelon of nodes for metastases if FNA positive for melanoma. 3. No prophylactic lymph node dissection should be offered without an attempt first at sentinel node biopsy. This now includes the head and neck. All intermediate depth melanomas, Breslow depth 1-4 mm, should be offered sentinel node biopsy. Thin melanomas with 3 of following risk factors men, ulcerated, truncal lesion, presence of regression, Clarks level of III or greater, high mitotic rate, nodular pathology might also be offered SNB. Consider use of SNB in melanomas thicker than 4 mm; unlikely to affect survival rate but might enhance local control. Utility of SNB in subungual melanomas not certain. Board answer is to complete node dissection for any positive sentinel node, although in presence of low level nodal involvement many surgeons are simply following the nodal basin with ultrasound. Get preoperative radiolabeled lymphoscintigraphy to define drainage pattern of the primary (this is especially important in truncal melanomas, whose drainage patterns are not predictable); can then either intraoperatively localize sentinel node with isosulfan blue and/or handheld gamma counter. Use of blue dye in melanoma less advantageous as the tracer identifies the 1 or 2 nodes involved with high rate of ID. As noted above, safe answer is to complete node dissection if sentinel node positive, although in >85% of cases it is the only positive node; if negative, close and wait for the final path. Sentinel node is evaluated with multiple serial microsections and immunohistochemistry (use of RT-PCR is being evaluated in the most recently opened Sunbelt Melanoma Trial and MSLT but is not standard of care).

page 22

GS-SDS-Oral 2010

GS_SDS_Oralrevised03012010 (6/2/2010)

The Osler Institute


4. Acceptable margins of excision of primary tumor: 5 mm for in-situ lesions, 1 cm for <1 cm depth, 2 cm for all others. No need for 4 cm margins even for very deep lesions. In cosmetically sensitive areas, may need to cheat down to microscopically clear margins. Margins used have no effect on survival, only local recurrence. 5. Rates of complications are lowest for head and neck dissections, greatest for groin dissections. Combining superficial and deep inguinal node dissections has >50% morbidity including extraordinary leg edema, seromas and recurrent cellulitis so be sure indications for deep dissection are rational. No need for deep dissection if only microscopic disease in superficial nodes. Consider deep node dissection if have gross disease in apical nodes (usually sapheno-femoral or Cloquets nodes), superficial involvement in more than 3 nodes, one node >3 cm in size, or if CT/PET scan shows suspicious iliac adenopathy. However, if CT suggests that nodes are positive to the level of the aortic bifurcation, iliac lymphadenectomy is unlikely to have any therapeutic benefit since there is almost certainly more proximal disease that will not be resected. Also do not do deep node dissections in patients with high number of medical comorbidities (competing causes of mortality) or very high risk primaries (e.g. an ulcerated 8 mm deep nodular melanoma in a 78 years old man) as resection of lymph nodes will not influence their over 70% chance of manifesting distant metastases. 6. Interferon alfa-2b is offered as adjuvant therapy to patients with melanomas deeper than 4 mm and no distant metastatic disease or those with Stage III disease. Given high-dose intravenous for one month, then 11 months of three times weekly subcutaneous injection. 7. Treatment of Stage IV melanoma is an unsettled issue. Isolated metastases to the liver and lung may be resected for some survival benefit, and bowel metastases should be resected since many of these patients will live up to two years after presenting with a bowel obstruction. Once patient has been rendered free of gross disease, consider enrollment in vaccine trial. 9. Chemotherapy for melanoma fairly limited. DTIC is most active single agent, although cis-platinum seems more effective when combined with interferon and interleukin2. Chemoimmunotherapy with cis-plat/IFN/IL-2 said to have 10-20% complete response rate with rare survivor
GS_SDS_Oralrevised03012010 (6/2/2010)

GS-SDS-Oral 2010

page 23

Surgery Review Course


beyond three years. Recent interest in combining taxane with the tyrosine kinase inhibitor Sutent in metastatic melanoma. 10. Role of vaccines uncertain. All remain on protocol. Various forms of vaccines available: autologous whole cell, allogeneic cell lysate, allogeneic whole cell, GM-2 (carbohydrate antigen found on almost all melanoma cell lines) 11. Treatment of in-transit metastases is initially surgical excision with narrow margins. Other alternatives include intralesional injection with interferon or BCG, ablation, or imiqiumod cream. If the disease is so extensive as to be limb-threatening, offer limb perfusion or, if available, limb infusion.

SPECIAL SUBTYPES OF MELANOMA


1. Anal melanoma is almost uniformly fatal. No evidence that APR should be done except if lesion has invaded the sphincter muscle and patient has severe pain or is incontinent. Consider any treatment as palliative, so local excision is all that is usually needed. No role for SNB, interferon, node dissection, or chemo-immunotherapy outside of a research center. 2. While traditional wisdom is that subungual melanoma is associated with acral lentiginous pathology, almost as many of these lesions are nodular melanomas. of these involve either the thumb or great toe. Unlike other melanomas, depth of primary is not as predictive of outcome as TNM stage. Almost half of patients will have lymph node involvement. Split or lift nail to biopsy these. If finger involved, can amputate distal phalanx only as long as interphalangeal area is not involved; for all toes, Ray amputation since it is better to weight-bear on the metacarpal than on phalanx. Value of sentinel node biopsy unknown for these relatively rare melanomas, so conservative answer is to do elective inguinal or axillary lymphadenectomy, unless the primary is very superficial (< 1 mm), which is generally not the case. If nodes are found to be positive, some centers have reported the adjuvant use of limb perfusion but, again, no prospective trial has been (or will ever be) done to answer questions as to the efficacy of perfusion. 3. If given a melanoma presenting as an involved node with an unknown primary, do extensive staging work-up to rule out disease elsewhere. If this is negative, treat the
page 24 GS-SDS-Oral 2010
GS_SDS_Oralrevised03012010 (6/2/2010)

The Osler Institute


patient as Stage III patient do therapeutic node dissection and offer protocol adjuvant therapy.

GS_SDS_Oralrevised03012010 (6/2/2010)

GS-SDS-Oral 2010

page 25

Surgery Review Course

page 26

GS-SDS-Oral 2010

GS_SDS_Oralrevised03012010 (6/2/2010)

The Osler Institute

Adult Cardiac Surgery


Warren D. Widmann, MD Columbia University

I. Coronary Artery Disease


A. Anatomy of significance
1. (R) coronary artery supplies the SA node. 2. (R) coronary artery supplies the AV node. Surface coronary veins drain to the coronary sinus and right atrium. Deeper veins drain to the atria and ventricular cavities. Oxygen extraction by the myocardium is 75% of delivered O2 (vs body average of 30%). Increase O2 to heart thus requires increasing flow to myocardium. Blood flow to the myocardium occurs mostly in diastole. Obstructive CAD in order of frequency. (L) Coronary > (R) coronary (L) LAD > circumflex 40-50% has triple vessel disease. 80-90% has surgically correctable disease. Obstruction of a coronary artery has to be > 75% cross-sectional area to be physiologically significant. Results of CABG for 1 and 2 vessel disease >>marginal survival increase. 3. Definite improvement in survival is for 3 vessel disease and for left main disease. 4. Complication of myocardial infarctions requiring surgical Rx a. Ventricular rupture b. Acute septal rupture with VSD, left to right shunting (most common in women with anterior MI c. Papillary muscle rupture with acute mitral insufficiency (most common with posterior myocardial infarctions.) d. Ventricular aneurysm (late effect) with 1) Embolization (rare) 2) Arrhythmias (common) 3) Failure (common) 5. Surgical complications of CABG a. Cardiac tamponade b. Tension pneumothorax c. Atelectasis d. Phrenic nerve paralysis (? related to cold slush) e. Sternal wound infections f. Pancreatitis g. Expected mortality of good risk patients < 4% 1) Graft patency a) Saphenous vein @ 10 yrs 50% b) Internal mammary @ 10 yrs 85%
GS_SDS_Oralrevised03012010 (6/2/2010)

GS-SDS-Oral 2010

page 27

Surgery Review Course B. Pacemakers


1. Indications a. Sick sinus syndrome not well controlled with medication b. 2nd and 3rd degree heart block (Mobitz II) c. Symptomatic bradycardias d. Best results with dual chamber pacemakers 2. If patient has atrial fibrillation, it is an absolute contraindication to dual chamber pacing. Usual route of insertion of electrodes is via subclavian or cephalic veins, either side can be used. Insertion requires testing for threshold of stimulation satisfactory range is about 0.5-1.5 volts, 0.5-1.5 mamp for atrium as well as testing for satisfactory sensing in atrium, 1-3 millivolts, and 4-12 millivolts in ventricle. 3. Possible complications include a. Electrode dislodgement with loss of capture and/or sensing b. Myocardial perforation c. Access site thrombosis d. Infection e. Exit block unexplained rise in threshold without obvious dislodgement f. Electrode fracture g. Premature battery depletion 4. Cardiac assist devices a. Intra-aortic balloon pump (IABP) b. Balloon placed into thoracic aorta deflates during systole, inflates during diastole diastolic augmentation. 5. Indications a. To stabilize unstable angina patient preop or preangioplasty b. To support postop low output syndrome (post-CABG) c. To support patient with post MI VSD or papillary muscle rupture d. To support preop cardiac transplant patient briefly (VAD is better). e. To support patients at high risk for MI requiring emergent noncardiac operations 6. Complications a. Arterial insertion site injury with proximal subintimal dissection b. Distal embolization or occlusion c. Local infection, aneurysm formation 7. Specific cardiac lesions a. Aortic stenosis usually calcific b. Normal valve area 3 - 4 CM2
page 28 GS-SDS-Oral 2010

GS_SDS_Oralrevised03012010 (6/2/2010)

The Osler Institute


c. Operative indications < 1 CM and/or gradient Ao valve > 50 mm Hg 8. Aortic insufficiency a. May be associated with ASCVD or rheumatic fever as with AS, but also with: Marfans syndrome Endocarditis Aortic dissection Aortic trauma b. Mechanical valves last longer but require systemic anticoagulation. c. Mitral stenosis usually secondary to rheumatic fever. Long latent period d. Symptoms: CHF (Atrial fibrillation may produce CHF acutely) Embolization from left atrial clot Hemostasis Like AS (normal valve > 3 cm2, symptoms with 1 cm2) Like AS, mechanical valves post require post-op systemic anticoagulation, risk of 1%/yr mortality, but last longer than bioprosthesis.
2

C. Pericardial disease and trauma


1. Cardiac tamponade a. Acute: No dilatation can occur with 100 cc blood or clot. b. Chronic: massive dilatation, 1-3 liter possible, with enlarged cardiac silhouette 2. Signs and symptoms: a. Shock with tachycardia, paradoxical pulse, cyanosis, neck with vein distention, distant heart sounds 3. Diagnosis a. Suspicion! b. Low voltage EKG c. ECHO d. Pericardiocentesis may give false negative if blood is clotted, false positive if cardiac chamber aspirated. 4. Treatment a. Pericardiocentesis b. Open drainage Sub-xiphoid Left thoracotomy Thoracoscopy 5. Trauma a. Stab wounds, usually (R) ventricular b. Blunt rupture-aorta, rarely atrial rupture 6. Facts to remember: With proper myocardial protection, extracorporeal pump/bypass of 2-4 hours is usually well tolerated
GS_SDS_Oralrevised03012010 (6/2/2010)

GS-SDS-Oral 2010

page 29

Surgery Review Course


Air embolism is a risk of bypass and may be mimicked by calcific particle embolization. CABG gives good relief to 85-90% of survivors, but it improves survival only (L) in main and triple vessel disease patients. Chest pain, fever, tachycardia, pericardial or pleural rubs, post-CABG, with lymphocytosis, 2-4 weeks postop usually have the postpericardiotomy syndrome and respond to anti-inflammatory drugs. Equalization of pressures (R) and (L) atrium, RV diastolic and PA diastolic is diagnostic of cardiac tamponade. Cardiac neoplasms are most frequently metastatic. Most common primary cardiac neoplasm is atrial myxoma, found mostly in women in the left atrium. Embolization is common as is fever. Open cardiac massage for cardiac arrest with thoracic aortic clamping, is useful following blunt and penetrating chest injuries, but is rarely successful following blunt abdominal trauma, and those resuscitated successfully from a cardiac viewpoint have dismal overall outcomes.

page 30

GS-SDS-Oral 2010

GS_SDS_Oralrevised03012010 (6/2/2010)

The Osler Institute

Pediatric Cardiac Surgery


Warren D. Widmann, MD Columbia University

I. Acyanotic Heart Disease


A. Patent ductus arteriosus
1. One of the first successfully treated congenital heart lesions 2. W/o correction 40-50% died of SBE (pre antibiotic era) 40-50% of CHF 3. Indomethacin may induce neonatal closure. Transcatheter ablations possible 4. Operation can include clipping, ligation, or division. 5. Complications include hemorrhage, chylothorax, left recurrent nerve injury, phrenic nerve injury, phrenic nerve injury, and recurrence of PDA.

B. Coarctation of the aorta


1. Typically at the site of the obliterated ductus arteriosus. Remember neonate has patent ductus, and early postdelivery femoral pulses may be present with flow from ductus. 2. Proximal, upper body hypertension with distal hypoperfusion typical. Can be managed by balloon dilatation, but surgical resection with primary end to end anastomosis is the standard treatment. 3. Postoperative paraplegia is a rare complication. Persistent paradoxical hypertension may occur of questionable etiology.

C. Atrial septal defects


1. Secundum defects most common (primum defects involve A-V tissue). Usually shunt is L to R and patients are acyanotic. 2. Untreated life expectancy 40 years 3. Operation advised if pulmonary flow is 1.5 X the systemic flow. Closure is by primary suture or patch. Care to avoid obstruction of pulmonary vein orifices.

D. Ventricular septal defects


1. May be single or multiple 2. May be near conduction bundle 3. Produce left to right shunts 4. May have associated PDA, coarctation of aorta 5. Symptoms more likely than with ASDs. Occasionally close spontaneously. Operative repair is done to prevent development of pulmonary hypertension. Patch repairs are done with care to avoid injuring conduction bundle posteriorly.

GS_SDS_Oralrevised03012010 (6/2/2010)

GS-SDS-Oral 2010

page 31

Surgery Review Course E. Cyanotic heart disease


1. Tetralogy of Fallot, most common a. VSD b. RV outflow obstruction c. Over-riding aorta d. RV hypertrophy 2. Boat-shaped heart. Formerly treated with BlalockTaussig shunts (subclavian to pulmonary artery shunting creating an artificial PDA now treated by complete repair 3-6 months of age

F. Tricuspid atresia
1. Usually has associated defect to allow R to L shunting for initial survival

G. Truncus arteriosus
1. Takes a few weeks for cyanosis to develop 2. Early total repair recommended

H. Transposition of the great vessels


1. Has to have some connection (ASD or VSD) for initial survival 2. ASD 60-70% 3. VSD 25% 4. Prior: complex Mustard procedure and Senning procedures with redirection of flow by patches; now replaced by arterial switch procedure in 1st two weeks of life.

I. Valvular lesions
1. Aortic stenosis a. Symptomatic with > 50+ cm Hg gradients b. Typically have bicuspid valves c. Repair is by valve reconstruction, rarely by prosthetic valve placement. 2. Pulmonary valve stenosis a. Common defect b. Symptoms with > 75 + cm Hg gradients c. Balloon valvulotomy currently procedure of choice, except in severely ill neonates with critical stenosis. 3. Ebsteins anomaly a. ASD and abnormal tricuspid valve which causes severe tricuspid insufficiency.

J. Facts to remember
1. Fetal circulation causes a right to left shunt via the ductus and via the foramen ovale. 2. Obstructing lesions cause increased ventricular work. 3. R to L shunt = cyanotic shunt 4. L to R shunt = a cyanotic shunt a. PA flow 1.5 to 2.0 times systemic flow is physiologically significant. 5. Ostium primum ASDs are associated with Down syndrome.
page 32 GS-SDS-Oral 2010
GS_SDS_Oralrevised03012010 (6/2/2010)

The Osler Institute


6. Ostium secundum ASDs are twice as common in girls. a. Cyanosis is a contraindication to repair of a PDA. b. Qp/Qs is the symbol for pulmonary flow/systemic flow. c. VSDs are frequently associated with AS, MS, and coarctation of the aorta. 7. The Blalock-Taussig shunt connects the subclavian artery to a pulmonary artery, end to side. 8. Transposition of the great vessels is the most common cause of cyanosis in the first week of life. 9. Anomalous origin of the left coronary artery from the pulmonary artery results in a coronary steal syndrome. It can be treated by ligation of the anomalous artery, but bypass grafting is preferable along with ligation. 10. Normal ductus closure is initiated by a fall in pulmonary artery pressure with flow of oxygenated blood towards the lung from the aorta. 11. Complications of cyanotic heart disease include cerebral thrombosis, septic infarctions, and peripheral air embolism from venous infusions with small otherwise insignificant air bubbles.

GS_SDS_Oralrevised03012010 (6/2/2010)

GS-SDS-Oral 2010

page 33

Surgery Review Course

page 34

GS-SDS-Oral 2010

GS_SDS_Oralrevised03012010 (6/2/2010)

The Osler Institute

Pulmonary: Benign & Neoplasia


Warren D. Widmann, MD Columbia University

I. Benign and Neoplasia


A. Pulmonary diseases
1. Benign neoplasms Most frequent solid tumor is a hamartoma; usually slow growing, can be in any location, and may have calcifications which look like popcorn. It can be managed by needle aspiration biopsy; if any doubt wedge excision by thoracoscopy. Excise any that show progressive growth. Any density should be considered for possibility of AV malformation. CT with dye aids diagnosis. Not usually a surgical lesion; can be treated with embolization or coils, radiologically. 2. Pneumothorax Frequent in 18-30 year old, thin, tall. Spontaneous (apical cyst rupture). Disease is bilateral, not seen on plain x-ray. Synchronous bilateral collapse is rare (2%). Metachronous collapse is fairly common (15%). Any traumatic pneumothorax seen on chest x-ray should get chest tube. Minor pneumothorax found incidentally on CT for chest/abdomen trauma may be followed conservatively unless patient going to OR (risk then of positive pressure causing tension pneumothorax). Tension pneumothorax is a clinical diagnosis. X-ray may confirm, but emergency treatment may be necessary by Angiocath decompression second intercostal space, mid clavicular line, top border of rib. Spontaneous pneumothorax with persistent air leak (> 4 days) should have resection of apical cysts/blebs and pleurectomy. Any recurrent pneumothorax should have definitive treatment as well. 3. Pleural effusions Tap for diagnosis Effusion vs exudate? Specific gravity > 1.016 Protein > 3.0 Exudates generally require chest tube evacuation. Effusions may be managed by aspiration. 4. Malignant effusions a. Tap for diagnosis b. Pleural biopsy has greater yield than fluid cytology. c. Recurrences can be managed by sclerosis (Doxycycline, Quinine, Bleomycin, Talc).
GS_SDS_Oralrevised03012010 (6/2/2010)

GS-SDS-Oral 2010

page 35

Surgery Review Course


d. Pleurectomy rarely indicated for malignant effusions. e. Empyema f. Open drainage by thoracotomy being replaced by thoracoscopic drainage and debridement. g. Etiology 1) Pneumonia 2) Postsurgical 3) Extension from subdiaphragmatic process

B. Neoplasms
1. Uncertain behavior Bronchial adenomas (carcinoids) Common in younger patients than lung cancer Typically endobronchial and have a history of recurrent obstructive pneumonias, may have local wheezing and minor hemoptysis 10-15% metastasize to regional nodes Atypical carcinoids have 60-70% 5 yr survival. Typical carcinoids have 95% 5 year survival. 2. Malignant tumors a. Lung cancer Most common cause of cancer deaths in both men and women 95% occur in smokers. 3. Cell types 20% small cell (not usually a surgical lesion Rx with chemo and RT) 80% non-small cell carcinoma a. Types: Squamous 30-40% Adenocarcinoma 30-50% Large cell carcinoma 5% Bronchoalveolar (a sub-type of adenocarcinoma frequently multifocal) b. Classification by TMN is important. Only stages I, II are well recognized as appropriate for resection in most cases (given adequate cardiopulmonary status). 1) Stage I (TI, T2, N0), tumors without chest wall extension, more than 2 cm from carina. No nodal mets No distant mets 2) Stage II (T1, T2, N1) primary as in stage I. Nodal mets only: peribronchial or ipsilateral hilum (N1) No distant mets 3) Stage III (T1-4, N1-3) is controversial. IIIa (T1-4 N1-2) is surgical, and includes tumors abutting chest wall and even locally invading chest wall or pericardium (but not through
page 36 GS-SDS-Oral 2010

GS_SDS_Oralrevised03012010 (6/2/2010)

The Osler Institute


pericardium). Pancoast tumors (superior sulcus tumors) are in this group. IIIb, however, is not usually considered surgical (N3 disease contralateral nodes, scalene nodes) or mediastinal tumor growth (T4) or positive malignant effusion. 4) Rough survival statistics 5 year I 70-80% II 40% IIIA 25% IIIB 5% IV 5% 5) Lung tumor syndromes: Horners Apical tumors Hoarseness Recurrent nerve palsy typically L side Gynecomastia Adenocarcinoma Hyponatremia Small cell carcinoma Clubbing Adenocarcinoma and squamous Cushings Small cell carcinoma Hypercalcemia Squamous cell cancer

C. Facts to remember
1. Mortality of lobectomy 5% 2. Mortality of pneumonectomy 8-12% Complications of chest surgery, bronchopleural fistulae, especially after right pneumonectomy. Atrial fibrillation, especially after any resection in the elderly High risk for resection: FEV1, less than 800 cc unacceptable; FEV, 800-1000 cc likely will need initial postop vent; PCO2 > 50 also a grave concern. Left main stem bronchus is longer than right main stem bronchus. For cases of massive hemoptysis use rigid bronchoscope. Most common complication postresection of lung tissue is retained secretions and atelectasis. Most common chest wall bony tumor is myeloma, a harbinger of systemic disease. Most common chest wall tumor in young men is chondrosarcoma. Mesothelioma is caused by asbestos exposure. The risk is magnified by smoking. Of all lung cancer pts, overall 5 yr survival is 10%. Resection of pulmonary mets should be done only if the primary tumor site is controlled, there are no mets to other organs and can be done even if there are multiple lesions involving both sides providing complete resection can be tolerated. Wedge resections are best for mets.
GS_SDS_Oralrevised03012010 (6/2/2010)

GS-SDS-Oral 2010

page 37

Surgery Review Course

page 38

GS-SDS-Oral 2010

GS_SDS_Oralrevised03012010 (6/2/2010)

The Osler Institute

Vascular Surgery: Aortic


Warren D. Widmann, MD Columbia University

I. Aortic Disorders
A. Thoracic aorta
1. Common causes a. Arteriosclerosis > aneurysms b. Marfan Syndrome > aneurysms/dissection c. Hypertension > aortic dissections not a true aneurysm d. Trauma > aortic transection 2. Aneurysms a. Mycotic 1) Salmonella 2) E. coli 3) Enterococcus b. Traumatic Usually just beyond ligamentum arteriosum c. Syphilitic Ascending aorta d. Dissecting 1) Originating in aorta proximal to the ligamentum arteriosus, type A 2) Originating distal to the ligamentum type B a) Both can extend proximally as well as distally. b) Rx for dissections Alpha blockade followed by beta blockade 3. Type A a. Surgical Rx b. Replace or insert stent. 4. Type B a. Surgery if medical management complicated by failure of BP control, continued pain, size increase, development of CNS problems or visceral or extremity ischemia 5. Thoracic aneurysms risk of rupture great after size above 10 cm (vs 5.0 cm for AAA) 6. For Marfan, replace aortic root if > 6 cm. 7. Proximal extension of aortic dissection may rupture into pericardium causing tamponade. 8. Thoracoabdominal aortic aneurysm repair carries risk of paraplegia. No methods of repair are 100% effective in preventing it. Short cross-clamp times are helpful.

B. Abdominal aortic aneurysms


Elective repair if over 5 cm Procedure: tube graft or aorta iliac graft (depends on outflow). 1. Ruptured aneurysms (AAA)
GS_SDS_Oralrevised03012010 (6/2/2010)

GS-SDS-Oral 2010

page 39

Surgery Review Course


a. Control above celiac initially, then infrarenal after better exposure. 2. Complications a. Cardiac events b. Hemorrhage c. Hypotension on clamp removal d. Distal embolization e. Ureteral injury f. Renal failure embolic or ischemic g. Colon ischemia 3. Prosthetic graft infection

C. Facts to remember
1. Thoracic aortic aneurysms are most common in the descending aorta. 2. 3:1 male predominance 3. Marfan syndrome is an autosomal dominant disorder. 4. Acute dissection of the thoracic aorta is more common than rupture of AAA. 5. The best test for aortic dissection is the quickest one available once dissection is suspected. It can be a contrast CT, MRI, aortogram, or transesophageal echo. 6. Traumatic aortic transection is 80-90% fatal in the field. 7. 95% of AAA are infra venal. AAA is present in 8% of patients with unilateral popliteal aneurysms, and 33% of patients with bilateral popliteal aneurysms. 8. The most common means of detection of AAA is some kind of imaging study performed for another reason. 9. Best diagnostic means for details of AAA in patients at risk for toxic effects of contrast media is MRI. 10. Both hypertension and COPD are risks indicators for rupture of AAA. The national average risk of surgical repair for AAA, nonruptured is 5%. (Some series report 1%). 11. The risk of death remains 50% for ruptured AAA. 12. Ischemic colitis develops in 2-10% of cases following AAA repair. It is higher in AAA repairs than following aorta-iliac reconstruction. Colon ischemia increases risk of death 50-90%. The thoracic aorta adventitia supplies 60% of the wall strength. 13. Traumatic aortic transections may show widening of the mediastinum, blunting of the aortic knob, pleural capping of the left apex, depression of the left bronchus, esophageal deviation to the right as seen by NGT displacement, and fractures of left 1st and 2nd ribs. 14. The most common cause of death after recovery from AAA surgery is coronary artery disease. 15. The best test for serial observation of a small AAA is sonography. Aorta-enteric (duodenal) fistulization is a late complication of AAA repair.
page 40 GS-SDS-Oral 2010
GS_SDS_Oralrevised03012010 (6/2/2010)

The Osler Institute


16. Impotence is the most common nonlethal complication of AAA surgery. 17. Inflammatory AAA are rare (2-10%) recognizable grossly by being thick, white, and fibrotic, may have ureters pulled in medially and can be demonstrated by aortic wall thickening beyond the calcification seen on CT scanning, and are associated with elevated ESRs, which return to normal after surgery. No infectious etiology is known. 18. Groin masses after aortofemoral grafting are usually hematomas. 2% may be lymphoceles or lymph fistulae. Proximal lymphatic injury is likely. 19. Fistulization is best treated by re-exploration and ligation of the lymph source. 20. Lymphoceles may respond to aspiration and pressure dressings. 21. Spinal cord injury patients have a 2 x increased incidence of AAA. Colon ischemia is more common after ruptured AAA surgery than after elective AAA surgery. Risk factors include ligation of the IMA and stump pressures less than 40 mm Hg. Early postop BM, especially if bloody is a common sign. 22. Percutaneous transluminal angioplasty is best for short iliac artery lesions. 23. The blue toe syndrome is caused by microemboli from the heart or from anyplace in the aorta that has plaques. 30% arise from AAA. Distal aortic occlusion can occur by saddle embolization or by thrombosis of diseased vessels. 24. Transfemoral embolectomy or lytic therapy from above may be successful.

GS_SDS_Oralrevised03012010 (6/2/2010)

GS-SDS-Oral 2010

page 41

Surgery Review Course

page 42

GS-SDS-Oral 2010

GS_SDS_Oralrevised03012010 (6/2/2010)

The Osler Institute

Vascular Surgery: Venous


Warren D. Widmann, MD Columbia University

I. Vascular Surgery: Venous


A. Venous disorders
Primary varicosities disease of superficial veins and their valves Secondary varicosities disease of superficial veins and valves secondary to deep venous insufficiency Perths test (applying pressure to collapse the superficial veins) is positive if pain results from this maneuver with patient ambulation. If aching is relieved by elastic compression, Perths test is negative and the deep veins are patent. Primary varicosities tend to propagate from above down. Primary lower extremity VVs are best treated by compression stockings for symptomatic relief of aching. Most symptomatic cases may be treated by superficial saphenous ligation and stripping. Spider varices respond to sclerotherapy.

B. Venous thromboses
1. Acute superficial phlebitis Remove source if a veinous catheter was etiology (catheter removal). Excise vein if septic phlebitis. 2. Acute deep vein phlebitis a. Risks Pulmonary embolization Sequelae of venous insufficiency b. Extremity, DVT best diagnosed by Doppler flow studies and B mode sonographic imaging. c. Proximal DVT (ileo-femoral) has higher risks. Phlegmasia alba dolens may progress to phlegmasia cerulea dolens (cold painful cyanotic leg). d. Standard treatment is heparinization; clot lysis is reserved for more severe cases, and thrombectomy is rarely indicated. e. Anticoagulation: raise PTT 2x, and INR should be 2.0 once oral Rx is effective. Heparin therapy should precede Coumadin Rx to prevent worsening thromboses in antithrombin III deficiency patients. f. Postphlebitic veinous insufficiency results in medial and lateral supramalleolar ulcers. g. Pressure dressings, debridement, application of PDGF, and healing by granulation is best. Simple skin grafting usually fails.
GS-SDS-Oral 2010 page 43

GS_SDS_Oralrevised03012010 (6/2/2010)

Surgery Review Course


h. The Linton procedure, with ligation of feeding perforating veins may aid in skin grafting success. i. Insertion of deep vein valves by transplantation may aid patients with patent deep veins but insufficient valves.

C. Vena caval filters


1. Indications strong Recurrent PE in adequately anticoagulated patient For prophylaxis in patient with DVT and contraindication to anticoagulation Post pulmonary artery embolectomy 2. Indications relative Any case of major PE with risk of nonsurvival if any further emboli occur Cases with DVT propagation on therapy Cases with free-floating iliofemoral clots Filters are usually inserted into infrarenal vein position. IVC occlusion is rare, but if it occurs, fluids should be given rather than vasopressors. 3. Pulmonary embolization Dyspnea most common symptom. Hemoptysis is a late symptom. Tachypnea and tachycardia are present. A rub is rare (20%). ABGs hypoxia and hypocarbia CVP is elevated. Chest x-ray of little aid. Lung scan for screening. Rapid sequence CT with dye coming into use as better study. Rx is by anticoagulation. For patients with major embolization, lytic therapy indicated. Unstable patients may benefit from open embolectomy. Superficial migratory phlebitis may indicate underlying malignancy. Subclavian vein thrombosis Now, most common 2 to catheters May also result from thoracic outlet compression. Rx lytic therapy; surgery for TOS 4. Renal vein thrombosis Associated with nephrotic syndrome 5. Portal vein thrombosis In neonates 2 to umbilical vein catheterization Mesenteric vein thrombosis: secondary to hormone use, sepsis, dehydration, digitalis, and diuretics, and best Rx is anticoagulation. If bowel compromised, resection and second look may be necessary. Anticoagulation indicated

D. Facts to remember
In the upright man, venous pressure of the foot is greater than 100 mm hg.
page 44 GS-SDS-Oral 2010
GS_SDS_Oralrevised03012010 (6/2/2010)

The Osler Institute


Occlusive dressings decrease ambient oxygen tension and increase angiogenesis. Blood group 0 is protective against DVT. Blood group A increases the risk for DVT. Sequential compression boots and low dose heparin are equivalently effective in prevention of DVT. Warfarin is contraindicated in the first trimester of pregnancy because it is teratogenic.

GS_SDS_Oralrevised03012010 (6/2/2010)

GS-SDS-Oral 2010

page 45

Surgery Review Course

page 46

GS-SDS-Oral 2010

GS_SDS_Oralrevised03012010 (6/2/2010)

The Osler Institute

Endocrine Oral Board Questions


Charles D. Goldman, MD West Virginia University

Endocrine I. Thyroid
The Thyroid mass Have to be thinking of a hot nodule, MEN II, as well as about the more common sporadic thyroid cancers. Important points in patient history: radiation exposure (including that for treatment of Hodgkins disease and breast cancer), signs of hyper- or hypofunction of the thyroid, any history suggestive of pheochromocytoma or hypercalcemia; hoarseness or changes in the voice suggest anaplastic malignancy. Unless physical exam leaves you uncertain as to whether mass is cystic or solid, a situation where an ultrasound might prove helpful, FNA is the next step. Thyroid scan has little value save for the work-up of hyperthyroidism with a discrete mass. If MEN is suggested by symptoms or family history, obtain calcitonin, serum calcium and phosphate, urinary calcium, and urine studies for pheo. Get RET testing on all patients and families with MTC and MEN 2 features. In 75% of MTC that appears sporadic, role of routine RET testing not yet established. If well-differentiated thyroid lesion is suspected by FNA, obtain thyroglobulin prior to excising mass to see if can be used to monitor disease status. If confronted with follicular neoplasm as the FNA result, patient to surgery. Although age, primary size and capsular invasion criteria are being used in an attempt to identify those who qualify as having a low-risk well-differentiated thyroid cancer, almost all the schemes lack prospective predictive validation. Except for the smallest papillary thyroid cancers, total thyroidectomy is the procedure of choice. This allows radioactive iodine ablation to be effected with much lower doses of I-131, reduces the risk of local recurrence, and enhances the ability of thyroglobulin to be used as a marker for recurrent disease. Any nodes encountered are usually removed in a non-anatomic cherry-picking approach. In the demographically identified low risk groups, presence of nodal involvement does not negatively impact survival when dealing with papillary or mixed papillary-follicular tumors. For follicular or Hrthle cell cancers, total thyroidectomy is always done since the hematogenous spread of these more aggressive neoplasms mandates the use of radioiodine. If nodes are involved, which is rarely the situation, manage similar to papillary lesions. Medullary cancer is treated with total thyroidectomy, central node dissection and modified radical neck on the side of the primary mass. These tumors
GS_SDS_Oralrevised03012010 (6/2/2010)

GS-SDS-Oral 2010

page 47

Surgery Review Course


poorly accumulate iodine so not clear if any consistent value to radioablation. Evaluate for hyperparathyroidism prior to exploration of the MEN patient if encounter four-gland hyperplasia, remove three and implant one minced up in arm (not in neck as high rate local recurrence with medullary lesions can render neck a hostile environment to re-explore). Follow with serum calcitonin and DMSA scan for elevations. Anaplastic cancer usually fatal. Impingement on airway and/or esophagus is main problem. Total thyroidectomy if possible; if not, at least split the gland to relieve tracheal encirclement. Chemo and radiation have scant palliative effect.

II. Hyperthyroidism
Differential is Graves disease, nodular goiter, hyperfunctioning adenoma, and thyroiditis, especially early in its natural history. Workup with thyroid panel, measurement of LATS level in patients with features suggesting Graves disease (female, goiter, ophthalmologic findings), anti-thyroid antibodies if history of neck tenderness and hyperfunction suggest thyroiditis, ultrasound gland for dominant mass +/scan. Antithyroid medication is appropriate in disease states where clinical remission is possible, esp. thyroiditis. Ablation with I-131 is generally used only in the non-surgical candidate. Subtotal thyroidectomy is the treatment of choice for children and young women (cant be using antithyroid medication during pregnancy), patients with cardiac complications of their toxic thyroid, patient with a dominant mass which is either hot and non-suppressible, or in which you cannot rule out malignancy. To avoid thyroid storm intraoperatively, prepare patient for OR with PTU which is stopped 7-10 days before the operation. Some surgeons then start supersaturated iodine solution (Lugols) to reduce gland vascularity. If tachycardia remains a problem, add a betablocker preoperatively.

III. Goiter
Indications for subtotal thyroidectomy in the setting of nontoxic goiter are: inability to rule out malignancy, pain or cosmetic concerns, non-response to a year or more of thyroid suppression or a thyroid releasing hormone test that shows the goiter to be acting autonomously.

IV. Hyperparathyroidism
Remember the symptoms of hypercalcemia can be very vague: depression, abdominal pain, bone pain; some are more tangible such as kidney stones. Most common cause of hypercalcemia is bony metastases from epithelial malignancies such as breast and lung cancer, and multiple myeloma. Primary hyperparathyroidism is caused by solitary adenoma in 80-90% cases, four-gland hyperplasia in 10-15%, 2-3% have multiple adenomas, and
page 48 GS-SDS-Oral 2010
GS_SDS_Oralrevised03012010 (6/2/2010)

The Osler Institute


carcinoma causes the rest (here the calcium levels trend much higher and a mass is often palpable). Secondary hyperparathyroidism is four-gland hyperplasia seen in the setting of renal failure in which there is excessive urinary losses of calcium. Work-up of hyper PTH includes ruling out bone disease from cancer. Get direct assay of PTH and check it against the serum calcium on the normogram. Check urinary calcium to rule out familial hypocalciuric hypercalcemia. Cl/phosphate ratio > 33 consistent with hyper PTH. In borderline cases for surgery, get bone films to check for evidence of early osteopenia or bone cyst formation. Never operate unless absolutely certain of diagnosis due to risks of permanent hypocalcemia and recurrent laryngeal nerve damage. Indications for surgery: Persistent hypercalcemia. Surgery for the group with asymptomatic hyperparathyroidism remains a matter of great controversy. Increasingly surgery for hyperparathyroidism is minimalaccess or, at the least, is a directed open exploration. Sestamibi is injected preoperatively and abnormal gland(s) is localized intraoperatively with gamma counter. Assess gland and background radioactivity in manner similar to radioguided sentinal node technique. Accuracy of sestamibiguided surgery can be enhanced by measuring intraoperative PTH. If abnormal gland is removed, serum PTH should drop 50% or more from preoperative baseline within 10 minutes. If PTH does not drop this much, continue to explore. If only three glands are found, traditional wisdom is to remove the thyroid lobe on the side of the missing gland this is almost never successful in locating the missing parathyroid. Look in upper mediastinum, tracheoesophageal groove, carotid sheath, thymus, etc. If no luck, close up and restudy the patient in 4-6 weeks; if serum studies suggest retained gland, get another sestamibi localization study.

V. Islet Cell Tumors


A. Insulinoma
Manifests with hypoglycemic symptoms reversible with sugar intake. In absence of liver disease or hepatoma, this almost always represents insulinoma. Most are benign neoplasms. A fasting insulin/glucose ratio > 0.3 is diagnostic of an insulin-producing lesion. A dedicated helical CAT scan of the pancreas is sufficient to rule out unresectable metastatic disease and help to localize the primary. Even if the primary is not seen on the CT, get no further testing. Just explore the patient. At exploration, intraoperative ultrasound may be very helpful. Benign insulinomas may be enucleated from the pancreas, while malignant ones may require a more formal resection. Blind resections should not be done; transfer the patient to
GS_SDS_Oralrevised03012010 (6/2/2010)

GS-SDS-Oral 2010

page 49

Surgery Review Course


a facility that can perform rapid venous assays intraoperatively to detect the drop in insulin level with each subsequent resection. In the nesidioblastosis picture associated with MEN I, near total pancreatectomy may be needed.

B. Glucagonoma
Usually a bulky locally advanced lesion on presentation, often with metastases at time of diagnosis. Causes profound protein malnutrition, chelosis, fingernail changes reminiscent of iron deficiency anemia and a migratory dermatolytic rash. Surgery aims to debulk for palliation. Somatostatin may be useful to control symptoms.

C. Gastrinoma
Most common presentation is secretory diarrhea, not panulceration of the upper GI tract. Diagnose with baseline serum gastrin level >500 pg, paradoxic increase to 4-5X that within 30 minutes of secretin administration, and ratio of basal acid output to maximal stimulated output that is >0.6. As with insulinoma, get CAT scan to rule out Stage IV disease (much more common with gastrinoma than with insulinoma) then explore. If see extensive metastatic disease, might consider non-operative management with omeprazole, octreotide and perhaps some 5FU/adriamycin/streptozotocin. If metastases are resectable, make attempt at metastasectomy and do either parietal cell vagotomy or V and A to reduce gastric ulcer risk. If cannot find the tumor (look in duodenal wall), do parietal cell vagotomy and rescan patient in 6-12 months these patients generally have an excellent prognosis.

VI. Adrenal Pathology


A. Primary Aldosteronism (Conns Syndrome)
Symptoms of weakness, headaches, muscle cramps and polyuria. Will have mild sustained hypertension. Labs will show hypokalemia (due to excess production of aldosterone) and an elevated aldosterone/renin ratio. Need to differentiate between the single adenoma (75%) and bilateral adrenal hyperplasia. If aldosterone level drops after giving the ACE inhibitor Captopril, the patient has suppressible bilateral adrenal hyperplasia. Treatment is medical: spironolactone. To localize a suspected adenoma CAT scan is often adequate. If not selective venous sampling or NP-59 iodocholesterol scan may be helpful. These glands are easily removed via a posterior approach or laparoscopically, and surgery controls the disease in >90% of cases.

page 50

GS-SDS-Oral 2010

GS_SDS_Oralrevised03012010 (6/2/2010)

The Osler Institute B. Hypercortisolism


Cushings disease represents 3/4 of the cases of Cushings syndrome, a consequence of an ACTH producing tumor in the pituitary. The other 25% of syndrome cases are attributable to ACTH-like substances released from tumors (most notably lung, thymus, medullary carcinoma of the thyroid) or adrenal adenomata/hyperplasia. Get basal cortisol levels, serum and 24-hour urine, to confirm hypercortisolism. Next determine ACTH if decreased, autonomous production by adrenal adenoma; if it is increased, need dexamethasone suppression test Cushings disease will suppress, ectopic sites will not. Treatment is directed to the pituitary for Cushings disease, to the underlying malignancy in secondary Cushings and to the adrenal in ACTH-independent disease. Unilateral adrenal disease can be removed, but bilateral adrenal hyperplasia is best treated with medical adrenalectomy: aminoglutethimide and/or mitotane as bilateral adrenalectomy leads to Nelsons syndrome, hyperpigmentation and hypertrophy of the pituitary gland.

C. Pheochromocytoma
Symptoms of headaches, sweating, palpitations, and episodic/sustained severe hypertension. First obtain urinary measurements of VMA and metanephrine. Get calcium and calcitonin levels if family history suggestive of MEN IIA. If urinary values suggest pheo, get CT scan of abdomen and pelvis. MRI being used increasingly as study of first choice. I-131 MIBG may be helpful in delineating extra-abdominal sites of disease. Prep patient with two week of alpha blockade and aggressive fluid resuscitation. If tachycardia results, treat with beta-blocker. Explore the abdomen through a midline approach. Remember to fully visualize both adrenals as the disease is bilateral in 10% of cases; also assess the extra-adrenal areas for paragangliomata which tend to be norepinephrine producers. Be prepared for wide swings in blood pressure have nitroprusside, NeoSynephrine and phentolamine readily available, Removal of pheo has about a 90% rate of success in controlling the hypertensive episodes.

D. Incidentaloma
Burgeoning use of CAT for work-up has led to an increase in finding adrenal masses. First obtain history directed to adrenal hormone excess symptoms. If negative, get urinary catecholamines, cortisol and 17 ketosteroids, and serum electrolytes, to rule out function of the mass. If there is any evidence this is a functioning mass, it should be removed after appropriate work-up or preop preparation as noted above. Although there is some
GS_SDS_Oralrevised03012010 (6/2/2010)

GS-SDS-Oral 2010

page 51

Surgery Review Course


interest in using MRI T2 weighting to discriminate amongst adrenal masses 3-5 cm in diameter, a recent Mayo Clinic study concluded all non-functioning adrenal masses in excess of 3 cm diameter should be removed. All masses greater than 5 cm, regardless of function, should be removed.

page 52

GS-SDS-Oral 2010

GS_SDS_Oralrevised03012010 (6/2/2010)

The Osler Institute

Esophageal Surgery
Charles D. Goldman, MD Des Moines University/Mercy Medical Center Des Moines

I. Esophageal Questions for the Oral Boards


A. Motility disorders
Achalasia is the basically the only one ever asked. Slowly progressive dysphagia, progressing from solids to liquids, with nocturnal regurgitation of undigested food. Pain is less commonly seen with achalasia than with diffuse esophageal spasm. When working up dysphagia, UGI is always first testing done as it has no risk associated with its performance. In achalasia, shows dilated esophagus (sometimes presbyesophagus) with conical tapering (bird beak) of column at GE junction. Manometry is next done to confirm non-relaxation of LES with wet swallowing, high resting LES pressure, and disordered proximal peristalsis with non-propulsive tertiary contractions. Some observers feel that the presence of tertiary contractions is all that is necessary to diagnose achalasia in patient with suggestive symptoms and UGI. EGD follows to rule out esophagitis and also to R/O associated squamous esophageal cancer, which is 30-40x more frequent in achalasia patient. Initial treatment generally depends on who first sees the patient. Especially in the young and healthy, laparoscopic Heller myotomy with partial fundoplication of choice is the treatment. Pneumatic dilatation has reasonable longterm success but up to 5% rate of perforation. BOTOX can be useful in vigorous achalasia (motility generally preserved) but requires repetitive injection and causes esophageal fibrosis that can make subsequent surgery mor difficult.

B. Diverticula
Zenkers is a pulsion (false) diverticulum that presents with dysphagia, bad breath, gurgling in the neck, and regurgitation of undigested food. Will generally not be appreciated on physical exam. Barium swallow will demonstrate; do no further testing, especially EGD which might lead to perforation. The underlying problem in Zenkers is not the diverticulum, but upper esophageal striated muscular dyscoordination. All operations on Zenkers include cricopharyngeal myotomy, while the tic can either be left in place if it is small (< 3 cm) or it may by suspended from the pre-vertebral fascia for dependent drainage or resected if it is larger. Approach is usually via the left neck due to greater predictability of recurrent
GS_SDS_Oralrevised03012010 (6/2/2010)

GS-SDS-Oral 2010

page 53

Surgery Review Course


laryngeal nerve path on that side and fact that most Zenkers are found on that side. Epiphrenic diverticulum is also a false diverticulum that presents with dysphagia and chest pain. Usually seen in association with achalasia so treatment is both diverticulectomy and myotomy for the achalasia. Reinforce closure with pleural, intercostal muscle, or pericardial flap to reduce risk of leak.

C. GE reflux (type I hiatal hernia)


Many asymptomatic patients will reflux on UGI, so UGI is not diagnostic of reflux, although it does delineate the anatomy. Do EGD to confirm presence and severity of esophagitis, and to assess for Barretts. Manometry, although recommended by many, is not mandatory since the likelihood of occult achalasia is very low. 24 hour pH probes may be needed if the patient has symptoms highly suggestive of GERD with an EGD showing no esophagitis. Consider the possibility of GERD in adults presenting with late age onset of asthma Always consider nonoperative management first not just PPIs at double or triple usual dose, but smoking cessation, weight loss, avoidance caffeine as these measures may eliminate need for operation. Those who fail symptomatically or have stricture disease requiring repeated dilatation should be offered an antireflux procedure. Laparoscopic approach is done in >90% of these patient but postoperative dysphagia rates appear higher than with open approach. Taking down short gastrics and opening the posterior esophageal attachments may lead to less distortion of the GEJ and less dysphagia. If given shortened esophagus, while there are laparoscopic approaches that use a cardiac wedge approach to create a neoesophagus with stomach, may have to consider approach through the chest to create the gastric tube. If in chest, may be able to get away with a Belsey Mark IV 270 wrap without lengthening the esophagus. If in the abdomen, will need to lengthen the esophagus with stapler and do Nissen wrap around the neoesophagus. (CollesNissen procedure).

D. Type II-IV hiatal hernia


Presentation varies from dysphagia, GE reflux, anemia from ulceration in supradiaphragmatic stomach (Camerons ulcer) to asymptomatic one discovered incidentally on EGD or UGI. Not surgical emergency unless there is continuous pain suggesting incarceration/strangulation. Can repair from either chest or abdomen, though most surgeons now favor the latter using the laparoscopic approach. Reduce the hernia contents that may include colon or spleen in addition to
page 54 GS-SDS-Oral 2010
GS_SDS_Oralrevised03012010 (6/2/2010)

The Osler Institute


the stomach, excise the peritoneal sac, and close the dilated hiatus. Closing hiatus may require use of mesh, usually a biologic one to avoid erosion of permanent mesh. Since there is stretching of the mesenteric attachments of the stomach and volvulus of the stomach is associated with these forms of hiatal hernia, the stomach needs to be fixed in some manner. This can be done by either performing an antireflux procedure, by fixation to the arcuate ligament (Hill procedure) or by placement of an anterior gastrostomy (or some combination of the three).

E. Barretts esophagus
Although both intestinal metaplasia and gastric-type changes are considered Barretts, only the former is premalignant. There is no risk of malignant transformation in cardiac or fundic type Barretts, therefore no surveillance regimen is required for this type. While there is no question that Barretts intestinal metaplasia increases the risk of developing adenocarcinoma, no test at present can predict which patients will progress to adenocarcinoma. Suggestions for stratifying patient risk by use of DNA flow cytometry or genetic deletions assays yet to pan out. Surveillance is the key element of management, not antireflux surgery. For patients with long-segment Barretts but no dysplasia, surveillance endoscopy every 2-3 years is adequate. Those with short segment disease (<3 cm) need no surveillance if dysplasia is not present on two successive EGDs. Patients with persistent low-grade dysplasia require q 6-month surveillance, with four quadrant biopsies taken every 2 cm of Barretts mucosa. This is done twice and if there is no change, surveillance can be stretched out to annually. In the case of high-grade dysplasia, two pathologists should review the biopsies. If no carcinoma is encountered, eight weeks of an aggressive medical antireflux regimen is given and the patient is biopsied again. If the severe dysplasia persists, the Board answer is esophagectomy due to an approximately 25% incidence of coexistent adenocarcinoma in these patients. On the other hand, NCCN guidelines now state that for both Tis and T1a esophageal cancer, endoscopic resection or mucosal ablation is an alternative to esophagectomy, as long as preoperative endoscopic ultrasound does not suggest a deeper primary lesion or suspicious adenopathy. Of note, prior treatment for H. pylori eradication increases risk of proximal gastric adenocarcinoma and distal esophageal adenocarcinoma. The evidence for prophylactic use of fundoplication to prevent conversion to cancer in Barretts patients is still
GS_SDS_Oralrevised03012010 (6/2/2010)

GS-SDS-Oral 2010

page 55

Surgery Review Course


lacking; most of the cited evidence is from short term, single institution trials, although a cooperative VA trial was felt to be positive. At present, indications for a wrap remain symptomatic GERD in Barretts despite maximal medical therapy, stricture, or Barretts ulcer. Never wrap a patient with high-grade dysplasia as this may affect your ability to use the stomach as a conduit if esophagectomy is needed.

F. Esophageal perforation
Most common cause is iatrogenic, either bougienage or scopes. If leak is discovered early (Gastrografin UGI followed by barium if any suspicion of distal perforation; if negative and still suspect a leak, perform CT looking for secondary signs of leak such as pneumomediastinum or left pleural effusion. Careful EGD would be last option employed), primarily repair with buttress of pleura, pericardium, intercostal muscle, or, if the leak is distal, stomach. For perforations discovered late with diffuse mediastinal contamination, consider esophageal exclusion with drainage of the mediastinum and pleural space and diversion of the cervical esophagus. If the leak is proximal to an obstructing lesion, even if discovered late, mortality is lower if that lesion is definitively dealt with. In the case of an obstructing cancer, this means esophageal resection without primary reconstruction, cervical esophagostomy and gastrostomy tube placement; for the achalasia patient, Heller myotomy 180 degrees away from the site of perforation. Limited esophageal perforations, such as from a fishbone, where the contrast simply drains back into the esophagus rather than percolating out into the mediastinal tissue, may be considered for nonoperative management. Increasing use of esophageal stents to temporize in setting of limited and early leak with limited mediastinal contamination that would otherwise require operative drainage Cervical perforations generally present earlier and patients are less toxic than distal esophageal perforations. Many can be managed with a short course of NPO, TPN and antibiotics; some may require neck explorations simply to drain but not repair

G. Esophageal cancer
Although it is now standard of care, there is no sizeable prospective trial showing neoadjuvant chemoradiation followed by surgery is superior to surgery alone. First goal with the esophageal cancer patient is to exclude them as an operative candidate. Since palliation of dysphagia can be achieved with radiation, laser ablation and endoluminal stents, the need for palliative
page 56 GS-SDS-Oral 2010
GS_SDS_Oralrevised03012010 (6/2/2010)

The Osler Institute


resection/bypass has markedly diminished. Therefore, must first exclude stage IV disease (distant metastases, involvement of celiac or more distant nodes with distal cancers) or evidence of inoperability (tracheoesophageal fistula, invasion of aorta/heart). CT/PET is indicated in all esophageal cancer patients due to the high rate of disseminated disease. If these are present, do not offer operation, palliate with other methods. If patient still appears to be operative candidate (regional adenopathy only is not an exclusion criterion at most centers), assess depth of tumor and adjacent lymph node status with endoluminal ultrasound. If T1 lesion, take to surgery without preoperative treatment. If considering preoperative treatment for T2-3 or N1 disease, stage patient with laparoscope (place feeding jejunostomy tube at same time) and possibly thoracoscope. If negative for metastatic disease, then treat with five-week course of RT, with 5-FU and cis-platinum as radiosensitizers. Some interest in omitting radiation from preoperative regimen and this is especially true for distal esophageal adenocarcinoma where ECF (epirubicin-cytoxan-5-FU), a gastric cancer chemo regimen, can be used as primary treatment without radiation. Restage the patient 4-5 weeks later to again R/O stage IV disease. If still not metastatic (< 2/3 of patients in most series), offer esophagectomy. In spite of their various champions, cancer-specific survival rates are the same for Ivor-Lewis and transhiatal esophagectomy. Many favor the latter procedure due to ease of handling cervical anastomotic leaks (which can occur in nearly 40% of patients) versus a thoracic leak. Radical esophagectomy, which includes radical lymph node dissection of mediastinum and left neck, and resection of upper half of stomach in all cases has higher operative mortality and no discernable improvement in median survival despite its theoretical benefits. If primary lesion is T4 (invading adjacent structures such as heart, aorta, or trachea) on EUS, palliate by means other than surgery. Scheme above is generally for adenocarcinoma. Except for T1 lesions, squamous cell carcinoma of the esophagus is usually treated with chemoradiation only due to its much higher rate of response vs. adenocarcinoma

GS_SDS_Oralrevised03012010 (6/2/2010)

GS-SDS-Oral 2010

page 57

Surgery Review Course

page 58

GS-SDS-Oral 2010

GS_SDS_Oralrevised03012010 (6/2/2010)

The Osler Institute

Stomach for the Oral Boards


Charles D. Goldman, MD Des Moines University/Mercy Medical Center Des Moines

I. Stomach for the Orals


A. Peptic ulcer disease
1. No elective ulcer operations until you have proved H. pylori eradication 2. Must document NSAIDs use since if patients have complication of a duodenal ulcer, such as perforation or bleeding, all they may require is control of the bleed or perforation without any anti-ulcer procedure. Same situation when H. pylori status is unknown. 3. For recalcitrant duodenal ulcer disease that has failed maximal medical therapy and lifestyle modifications, the operation of choice is parietal cell vagotomy (highly selective vagotomy). This is essentially a lesser curve devascularization that spares the crows foot nerves to the pylorus. Carry dissection up at least 5 cm on the esophagus to lyse direct esophageal branches and divide criminal nerve as it emerges over the angle of His. This operation is contraindicated in patient with significant antral/prepyloric ulceration. 4. Vagotomy and pyloroplasty: Main role is in the unstable/elderly patient with an acute ulcer bleed. Its recurrence rate is little different from highly selective vagotomy, but it can be more quickly performed. Pyloroplasty alternatives are Heineke-Mikulicz, Finney (side-to-side gastroduodenostomy to the first portion of the duodenum) or Jaboulay (side-to-side to the third portion of the duodenum) 5. Vagotomy and antrectomy remains the gold standard for treating virulent ulcer disease, but at the cost of higher mortality and more long-term complications. Billroth I reconstruction is preferred with its only downside being a higher incidence of dumping. 6. In this era of PPIs and H. pylori eradication, there is less reason to be using V and A despite its higher level of ulcer prevention. The latter comes at the cost of many more complications and better functional results can be achieved by using a less effective anti-ulcer procedure, highly selective vagotomy, backed up by medical management. 7. Complications of gastric resections a. Dumping primary treatment is dietary modification, augmented with long-acting somatostatin. If this fails, surgical remedy is interposition of 5 cm jejunal loop (can be iso- or anti-peristaltic) between gastric outlet
GS_SDS_Oralrevised03012010 (6/2/2010)

GS-SDS-Oral 2010

page 59

Surgery Review Course


and duodenum (if Billroth-I in place) or conversion of Billroth-II to Roux-en-Y b. Postvagotomy diarrhea cause uncertain. Can try antimotility agents, antibiotics and bile acid binders such as cholestyramine. If this fails, reversed jejunal segment at 100 cm from ligament of Treitz c. Afferent loop syndrome emesis of food without bile, pain relieved with emesis. Either convert to Billroth I configuration, make into Roux, or decompress afferent loop into efferent loop with Braun enteroenterostomy. d. Bile reflux gastritis persistent epigastric pain not relieved by vomiting of food and bile. Can try Actigall and Carafate. If no relief, convert to Roux. e. Iron deficiency anemia EGD to rule out marginal ulcer. Check for iron deficiency may be profound due to bypass of duodenum with Billroth II configuration. f. Gastric carcinoma increased risk after latency period of 15-20 years with antiulcer operations. Start EGD surveillance around 15 year point. 8. Gastric ulcer treatment same as duodenal if ulcers are distal in stomach or have combined gastric and duodenal ulcers. These are hyperacidity-induced 9. For type I gastric ulcer, not related to hyperacidity, surgery for failure of medical therapy. Offer antrectomy with excision of ulcer. No need to add vagotomy. Biopsy all gastric ulcers prior to treatment to R/O malignancy, especially if > 2 cm in diameter.

B. Complications of peptic ulcer disease


1. Given availability of proton pump inhibitors and H. pylori eradication, need for definitive surgery at the time of acute presentation is waning. 2. For the most part, perforated duodenal ulcer only requires patch, unless patient has long history of ulcer disease unresponsive to medical treatment. 3. Perforated gastric ulcer should be excised. In unstable patient, plugging with omentum is safer than excision. 4. Bleeding duodenal ulcer should have at least two attempts at endoscopic control; if blood loss continues, make limited duodenotomy (allows you to reconstruct pylorus and still do a highly selective vagotomy) to oversew bleeder then offer definitive surgery only if sure ulcer is not NSAIDs related and/or the patient has been tested and treated for H. pylori previously. Definitive operation chosen using same criteria as elective OR detailed above. 5. Bleeding gastric ulcer is rarely controllable with endoscopy so only one attempt should be made. If fail, resective surgery based on ulcer type.
page 60 GS-SDS-Oral 2010

GS_SDS_Oralrevised03012010 (6/2/2010)

The Osler Institute


6. Most patients with stress gastritis will not require surgery. If they do, total gastrectomy has lowest rate of rebleed but takes time in unstable patient, gastric devascularization may be better choice in the unstable patient.

C. Gastric outlet obstruction


1. Virulent ulcer disease leading to pyloric obstruction 2. Present with prolonged and repetitive vomiting resulting in hypokalemic hypochloremic metabolic alkalosis 3. Fluid resuscitate and begin TPN 4. EGD to assess H. pylori status and rule out malignancy as cause of obstruction 5. Aggressive anti-acid treatment for 10-14 days, including H. pylori eradication 6. Vagotomy and antrectomy for failure of medical therapy a. Duodenal closure may be difficult due to inflammation so be prepared with alternatives for closure, especially the Nissen closure that externalizes the ulcer and uses surrounding inflammatory tissue to hold posterior layer of closure.

D. Gastric adenocarcinoma
1. Guidelines for treatment now firmly recommend preoperative treatment either with chemoradiation (5-FU, taxane, radiation) or chemotherapy alone (ECF) for all T2 and above gastric adenocarcinomas 2. Resect with 5 cm margin so if lesion within 5 cm of GE junction, patient likely needs total gastrectomy, although Japanese have published several series using proximal gastrectomy with the gap being bridged with a segment of jejunum or a Roux limb. 3. Only resect spleen if it is grossly involved with tumor. No survival benefit in its prophylactic removal. 4. If patient able to eat preoperatively, consider placing laparoscope at outset of procedure. If find liver mets or peritoneal implants, might consider not opening patient at all as resection will not improve quality of life. Obviously, if cancer is bleeding or obstructing, will need resection anyway so laparoscope not useful in those situations. 5. There is no evidence for Japanese style peritonectomy or extended lymph node dissections enhancing survival in large prospective randomized Western trials. Operative mortality is increased, effectively swamping any putative survival benefit. 6. May still be some role for postoperative adjuvant chemoradiation, generally 5-FU and radiation 7. No evidence for hepatic metastasectomy in gastric adenocarcinoma.

GS_SDS_Oralrevised03012010 (6/2/2010)

GS-SDS-Oral 2010

page 61

Surgery Review Course E. Gastric lymphoma


1. Another very controversial area where it is difficult to ascertain what the Board answer will be. Oncology guidelines are clear that gastric resection is only for salvage therapy after failure of medical and radiation treatment, but surgical texts still persist in recommending surgery first 2. MALT lymphomas are borderline lesions that may be either considered premalignant or low-grade nonHodgkins lymphomas. 3. All forms of gastric lymphoma are highly associated with the presence of H. pylori that induces lymphoid hyperplasia. 4. If given a patient with Stage I-II MALT lymphoma (the usual clinical and Board scenario), the treatment is H. pylori eradication with restaging endoscopy and biopsies at 3 months. If lymphoma negative, scopes continue at 3 month intervals for a year then go out to 3-6 months thereafter. If the lymphoma and the H. pylori persists, second line antibiotic therapy for H. pylori is administered. If the lymphoma persists but the bacteria is now eradicated, radiation therapy is given. 5. In the case of a more advanced MALT lymphoma or bulkier follicular lymphomas, chemoimmunotherapy (CHOP and Rituximab) is given. Radiation may be added in the case of non-responsive bulky disease, while surgery would be reserved only for failures of all the preceding treatments F. Surgery for morbid obesity 1. Most widely used operation in the U.S. is the Roux-en-Y gastric bypass, although the pendulum has begun to swing more in the direction of gastric banding, especially in those with BMIs < 50. A 15-30 cc pouch is made of the proximal stomach and a < 2 cm gastrojejunostomy is made to the pouch. An enteroenterostomy is created ~75150 cm from the gastrojejunostomy. 2. Average weight loss with RYGB is 50-70% of excess body weight by 2 years. Some weight gain after that time is common. 3. Gastric banding weight loss results are more modest, averaging 40-55% of excess body weight loss 4. Resolution of obesity associated disorders, especially diabetes, occurs more consistently and earlier with the gastric bypass 5. The increasing preference for banding reflects its relative technical ease of insertion and its very low rate of morbidity and mortality compared to RYGB
page 62 GS-SDS-Oral 2010
GS_SDS_Oralrevised03012010 (6/2/2010)

The Osler Institute


6. Generally agreed-upon criteria for offering the operation is: age < 65 years, body-mass index of greater than 40 with or without obesity-associated morbidities, or body mass index 35-40 with obesity-related morbidities, reasonable operative risk 4. Obesity-related morbidities include hypertension, diabetes, sleep apnea, severe degenerative joint disease, family history of young age of onset cardiac disease, pseudotumor cerebri 5. Operative mortality of RYGB is <1% with most deaths being secondary to either septic complications of anastomotic leaks or fatal pulmonary embolism.

GS_SDS_Oralrevised03012010 (6/2/2010)

GS-SDS-Oral 2010

page 63

Surgery Review Course

page 64

GS-SDS-Oral 2010

GS_SDS_Oralrevised03012010 (6/2/2010)

The Osler Institute

Colon Surgery
Charles D. Goldman, MD Des Moines University/Mercy Medical Center Des Moines

Oral Questions: Colon I. Large Bowel Obstruction


Overall the most common cause of large bowel obstruction is malignancy, especially in the older patient population. However, volvulus is the most common cause of right-sided colon obstruction, due to relative infrequency of obstructing right colon cancers. Cecal volvulus presents in a manner akin to small bowel obstruction, while the more distal colon obstructions manifest with constipation/tenesmus early, nausea and vomiting late. Adhesions and intussusception are rare cause of adult colonic obstruction. Diagnosis suggested by history, exam and 3-way abdomen. If still uncertain, barium enema can be used in absence signs perforation, otherwise use water soluble contrast (NB If suspect closed loop obstruction, as in volvulus, do not retrograde water soluble agents into closed loop as hydrophilic action worsens wall ischemia and can lead to perforation.)

A. Operating in the presence of acute obstructing cancer 1. Main issue is whether there is a need for bowel prep 2. For right sided lesions, primary resection is now generally agreed upon 3. On the left, primary resection is still an option. Stenting to allow prep can be considered. Other alternatives include resection with primary anastomosis protected by loop ileostomy, subtotal colectomy with ileorectal anastomosis, or two-stage procedure with an initial diverting colostomy B. Diverticulitis
Classic presentation of fever, left lower quadrant abdominal pain, and bowel dysfunction. Most cases can be treated with outpatient antibiotics; admit the elderly/immunocompromised/etc, for IV antibiotics and hydration. Most patients with moderate-to-severe diverticulitis will undergo CAT scan as initial study to rule out pericolonic abscess. If case remits with medical treatment, schedule barium enema and possibly colonoscopy to rule out a luminal lesion.

GS_SDS_Oralrevised03012010 (6/2/2010)

GS-SDS-Oral 2010

page 65

Surgery Review Course A. Indications for sigmoid resection


1. Lack of response to medical treatment 2. Complication of diverticulitis most commonly colovesical fistula 3. Inability to rule out malignancy 4. Chronic obstructive symptoms suggestive of inflammatory stenosis of the lumen 5. Recurrent acute episodes When operating for diverticulitis, not necessary to remove all of the tic-bearing colon, only the high-pressure zone of the sigmoid. Soft, pliable descending colon should be anastomosed to upper rectum demarcated by the disappearance of the colonic teniae

Diverticulitis surgery
1. Lots of controversy at present over the necessity for sigmoid resection in cases of uncomplicated diverticulitis or abscess that are percutaneously drained to a good outcome 2. One study out of U Washington showed that most costeffective approach is to operate after 7 episodes of diverticulitis, so traditional answer of operate after one episode in the young and two in the older patient is being called into question 3. Availability of laparoscopic colectomy has been impetus to try to cool off the patient so as to facilitate an easier laparoscopic resection. This has led to recent suggestion that acute episode should be treated with laparoscopic irrigation of left lower quadrant with drain placement. 4. Options for surgery are similar to that enumerated previously for an obstructing cancer, with the possible addition of on-table lavage to facilitate primary anastomosis. Perception that two-stage Hartmans procedure is safer ignores the high rate of complications associated with takedown of the colostomy, especially is steps arent taken to locate the rectal stump in an easily accessible position near the pelvic brim

C. Volvulus
1. Cecal: colonoscopic detorsion only for those with prohibitive operative risk, but still leaves uncertainty regarding irreversible ischemia. Best long-term results are with ileocecal resection; less acceptable alternative is cecopexy. 2. Sigmoid: sigmoidoscopic decompression, passage rectal tube. In young, good-risk patient, elective resection; in elderly, operate only for recurrence or acute abdomen.

page 66

GS-SDS-Oral 2010

GS_SDS_Oralrevised03012010 (6/2/2010)

The Osler Institute D. Pseudo-obstruction (Ogilvies syndrome)


1. Most common cause is opiate pain medications or patient immobility. Observe cecal diameter on plain films; check electrolytes including calcium and magnesium, thyroid functions; decompress with colonoscope or try IV neostigmine. In the acute setting, options are cecostomy versus right hemicolectomy, with the latter always used if question of ischemia. For the patient with chronic Ogilvies can consider placing a button cecostomy for periodic decompression.

III. Toxic Megacolon


Although usually associated in most physicians minds with ulcerative colitis, also seen with Crohns disease, amebiasis and, increasingly and ominously, C. difficile colitis. Often can diagnose on plain abdominal films which shows transverse colon dilation > 6 cm, wall thickening, and luminal air contours consistent with pseudopolyps. Try to avoid contrast studies as this can exacerbate the illness or lead to perforation. Patients are aggressively fluid resuscitated as fluid and electrolyte losses are profound, put on bowel rest, broad spectrum antibiotics, and high dose steroids if known to be an inflammatory bowel disease patient. TPN is begun once urine output is assured. Non improvement in 12-24 hours is indication for surgical exploration. If forced to operate, perform ileostomy and subtotal colectomy. Especially in the setting of ulcerative colitis want to preserve rectum for possible elective performance of ileoanal pullthrough procedure.

IV. Ulcerative Colitis


Nontransmural inflammatory bowel disease. Extraintestinal manifestations similar to Crohns, except sclerosing cholangitis is more common and anal disease less frequent. Various infectious entities (salmonella, shigella, legionella, HSV, chlamydia, CMV) can mimic UC so cultures and biopsies important. Chronic disease that is far more amenable to medical treatment than Crohns. Unlike Crohns where steroids are reasonably efficacious, maintenance treatment with sulfasalazine reduces relapse and suppresses symptoms. Other alternatives are 5-ASA, AZT, 6-MP, cyclosporine. Although overstated in earlier studies, cancer risk is main driver of follow-up scheme in UC. Within 5-7 years after initial diagnosis, patient should start undergoing annual surveillance colonoscopy to assess for dysplasia. With the first encounter with any degree of dysplasia, especially starting at the ten-year point, consideration should be given to prophylactic colectomy.

A. Surgical options in the elective setting include


1. Subtotal colectomy with ileorectal anastomosis if rectum reasonably free of disease (rarely the case in UC, but may
GS_SDS_Oralrevised03012010 (6/2/2010)

GS-SDS-Oral 2010

page 67

Surgery Review Course


be the case in indeterminate colitis). Best in younger patients with better sphincter tone and where the advantage of lower rates of postoperative GU dysfunction because of no rectal resection are most readily apparent. 2. Total proctocolectomy with creation of continent Kock pouch technically demanding with few surgeons skilled in creation of the continent ileal pouch. 3. Total proctocolectomy with rectal mucosectomy and ileal pouch-anal anastomosis-preferred approach at present time for most patients. This procedure has been made even simpler by omission of the mucosectomy. The rectum is removed and stapled closed just above the dentate line. The pouch is then stapled to the rectal stump using the standard two-staple technique. Only question is that this leave 2-3 cm of rectal mucosa in continuity which needs to be surveyed for possible development of cancer 4. Total proctocolectomy with Brooke ileostomy

V. Polyps
Questions revolve around two dilemmas: cancer in polyps and the familial syndromes. Although all the data is retrospective and not that voluminous, most surgeons agree that polypectomy is adequate treatment for a cancer if it is: well differentiated, does not involve the stalk of the polyp, makes up less than 50% of the volume of the polyp, and is nonulcerated. Invasion of the muscularis mucosa is not an absolute indication for segmental resection as long as the preceding criteria are met. Despite the Cronkhite-Canada syndrome (familial juvenile polyposis) and Peutz-Jeghers polyposis being hamartoma associated, they carry a small risk (< 10%) of colorectal carcinoma. The adenomatous polyposis syndromes (FAP, Gardners, Turcots) carry variable risk, ranging from 50100%. Although the mean age for the development of carcinoma in FAP is 40 years old, most of those identified with the syndrome undergo prophylactic colectomy prior to age 25. Options are similar to those with ulcerative colitis, since removal of the entire colonic mucosa is all that is necessary to eliminate the cancer risk. Most surgeons, however, do not offer subtotal colectomy with ileorectal anastomosis since the retained rectum still carries a substantial malignant degeneration risk. As with ulcerative colitis, the desirable Board answer is ileal pouch pull-through. Remember that if do the coloanal anastomosis with two staple technique, 2-3 cm of rectal mucosa above the dentate line is left in place and this mucosa remains at risk for malignancy.

VI. Colon Cancer


No role for preoperative therapy, unlike rectal lesions. Although no-touch technique itself does not appear to
page 68 GS-SDS-Oral 2010
GS_SDS_Oralrevised03012010 (6/2/2010)

The Osler Institute


enhance survival, its underlying principle, resection of colon based on named vessels supplying the region of the malignancy, assures an adequate lymph node dissection that is key to surgery for colon cancer. Five cm luminal margins are the standard but if do resection based on vascular supply will invariably get more than this distance. If the colon cancer is invasive into adjacent tissue/organs, do not separate tumor from the invaded structure even if this involves performing an extensive en bloc resection, eg, a Whipple resection. Dissecting tumor off leads to < 5% long-term survival even if tumor itself only Stage II. In women < 50 years old with colon carcinoma consider prophylactic oophorectomy, especially if past child-bearing years. This has been shown to enhance survival due to high rate of synchronous and metachronous ovarian metastases, and reduction in risk of primary ovarian cancer in patients with augmented risk for that diagnosis. No oophorectomy benefit seen in postmenopausal patients. Patients with poor risk Stage II disease (invasion into other organs, poorly differentiated tumor, perforated cancer) and all Stage III patients (lymph node involvement) should be given adjuvant chemotherapy, now FOLFOX (infusional 5-FU-leucovorin-oxaliplatin). Issue of following CEA so as to detect recurrence continues muddled it is clearly not cost-effective, costing about $300,000 for year of patient-life saved, but is probably still the Board answer. Make sure you check CEA before resecting the tumor to ascertain whether follow-up CEA will have any value.

VII. Hereditary Nonpolyposis Colorectal Cancer (HNPCC)


Patients are characterized by displaying microsatellite instability (MSI) with mutations in DNA mismatch repair genes (most commonly MSH2 and MLH1). These deletions can now be sequenced by same company that does BRCA testing. Amsterdam criteria (history of colorectal cancer in three generations, with two affected first degree relatives in two successive generations with at least one presenting prior to age 50) traditionally used to identify those who may have HNPCC. Some have advocated widening criteria to multiple family members in multiple generations, some with early onset of colorectal, other GI, or endometrial cancer, and multiple tumors (either synchronous or metachronous) in the same individual. If suspect HNPCC, obtain genetic testing and start to screen family members with colonoscopy annually starting at age 25. For females, consider endometrial cancer surveillance with pelvic ultrasounds and/or endometrial biopsy annually starting at age 25. As with BRCA, family testing is facilitated by first testing an affected family member to characterize the mutation you are looking for. Unclear whether patients who are found to have these
GS_SDS_Oralrevised03012010 (6/2/2010)

GS-SDS-Oral 2010

page 69

Surgery Review Course


deletions should undergo total abdominal colectomy or simply continue annual colonoscopy with polypectomy; no prospective evidence favors either approach. Females with HNPCC should have total hysterectomy after childbearing is completed, as there is no effective screening regimen for endometrial cancer.

page 70

GS-SDS-Oral 2010

GS_SDS_Oralrevised03012010 (6/2/2010)

The Osler Institute

Rectal Surgery
Charles D. Goldman, MD

I. Rectal Cancer
A. Staging system same as for colon cancer, prognosis slightly worse
Consider use of preoperative therapy in all but the most favorable stage I lesions (T1-2, favorable pathology). Other than biopsy of the mass, endorectal ultrasound is by far the most important test as it allows a reasonable estimation of depth of invasion and is slightly superior to MRI and CT at assessing involvement of pelvic lymph nodes (endorectal MRI may be the most accurate but is more uncomfortable for patient and not widely available). If rectal mass is fixed to surrounding structures, preoperative chemoradiation (5-FU weeks 1 and 5, with six weeks of radiation to around 5500 cGy) is administered in an attempt to downstage the tumor and render it resectable. Surgery is performed 2-4 weeks after the completion of radiotherapy. If the mass is not fixed, regardless of lymph node status, short course radiotherapy only can be used. 2500 cGy are given over 5 days with no chemosensitizer and surgery is performed 2-3 days after completing RT. Short course cannot be used with fixed lesions, while long course regimen can be employed in either situation. For unclear reasons, use of short course radiotherapy is very limited in the U.S. Minimum margin for safe resection of rectal tumor is generally stated to be 2 cm. Tangential margin is far more critical than the luminal margin consistent literature support for total mesorectal excision decreasing local recurrence and increasing survival. (Know how to describe key elements of mesorectal excision, especially that mesorectal dissection is carried at least 2 cm past the transection line onto the rectal stump. This causes higher leak rates, which is why some surgeons routinely protect their anastomoses with loop ileostomy after mesorectal excision) If appropriate margins are obtained, no difference in survival or recurrence between APR and low anterior resection. When LAR chosen for lesions < 5 cm from anal verge, functional results of colonic J-pouch or coloplasty are superior to straight reconstruction at one year, although they are comparable at longer follow-up. If patient has not received radiation prior to operation and is found to have a high-risk stage II or any stage III tumor, follow resection with six weeks of chemoradiation
GS_SDS_Oralrevised03012010 (6/2/2010)

GS-SDS-Oral 2010

page 71

Surgery Review Course


as described above and then 4-6 further cycles of 5-FU/leucovorin after that. If had radiation prior to OR, just follow with chemotherapy. Preferred colon adjuvant regimen, FOLFOX, is still not considered first-line as not adequate research experience with it; it however can be used in lieu of the more traditional regimen Can consider local excision of rectal tumors if: within 8-10 cm of anal verge (need to be extraperitoneal rectum and reachable with available instrumentation), well to moderately differentiated tumor without ulceration, not invasive into perirectal fat (T2 or less), no lymphovascular invasion on pathology, preferably exophytic, less than one-third rectal circumference. If do local excision and pathology returns Stage II depth of invasion, most centers would then add postoperative external beam radiation therapy to primary site and pelvis.

B. Anal cancer
Make sure they are describing a cancer of the anal canal, not a squamous cell of the anal margin. The latter are treated like any squamous cell of the skin using wide local excision. Only exception to this for SCC of the anal margin is extensive carcinomas of this region (e.g., Buschke-Lowenstein giant verrucous squamous cell carcinomas) that can be treated with chemoradiation regimens similar to that used for anal canal cancers rather than APR. Anal canal cancers are treated with chemoradiation. After confirmatory biopsy, 5-FU and mitomycin are given as radiosensitizers and the region is radiated with 4,500 to 6,000 cGy. Synchronous nodal involvement is treated by including these areas in the radiation field. Routine follow-up biopsy is no longer advised; biopsy only for suspicion of recurrence on digital rectal exam. If there is persistent tumor a small amount of further radiation can be given, this time using cis-platinum as an adjunct agent. If there is still no resolution, APR is performed for salvage. Groin dissection may be needed if recurrent disease there.

C. Anal melanoma
Local excision only, no role for APR as a curative operation. Only time to consider APR is patient who is yet to develop metastatic disease (a rare circumstance with anal melanoma, which has an abysmal prognosis) and has extensive and symptomatic local disease. No benefit can be inferred for interferon and consider node dissections only for gross nodal disease which, after a thorough search for other metastatic sites, appears to be an isolated finding.

D. Anal fissure
Conservative measures, analgesics and stool softeners, initially. If fail to heal, can try locally applied Nitro paste
page 72 GS-SDS-Oral 2010
GS_SDS_Oralrevised03012010 (6/2/2010)

The Osler Institute


or nifedipine ointment. Another alternative, and seemingly more effective than Nitro paste, is Botox injection into the anterior midline to reduce internal sphincter spasm that is felt to cause ischemia and the resultant fissure. If these measures fail to heal the fissure, perform lateral internal sphincterotomy. Small risk of involuntary incontinence.

F. Anal fistula
Main points to remember: Goodsalls rule only helpful if fistula openings are within 3 cm of the anal margin. Complicated recurrent fistula disease suggests one of two things: an undrained crypt abscess, usually in the posterior midline, or Crohns disease. Fistula disease may present years before the intestinal manifestations of Crohns (which is usually ileocolic, not colonic) so diagnosis can be difficult. Anal fistulas rarely contain granulomata, even if secondary to Crohns. If suspect Crohns is cause of the flowerpot perineum, can try p.o. Flagyl. Otherwise, appropriate fistulotomy should be done. Even in the presence of Crohns, conservative fistulotomy can be performed if one area is preferentially symptomatic and fistulotomy will not put too much sphincter bulk at risk. Setons can be used simply as drains or as cutting setons. Results equivalent with either type of seton. Consider their use when fistulotomy would lead to disruption of most of the sphincter muscle mass. This is especially true for anterior fistulae. Alternative to seton is fibrin glue injection to seal the fistula tract without having to cut fistulotomy. Worth trying as fibrin glue does not burn any bridges or cause morbidity. Some success with Surgisys plugs if can maintain in place for at least 2 weeks after surgery. Fibrin glue appears to have little value in Crohns-associated fistula, while Surgisys plugging may work.

GS_SDS_Oralrevised03012010 (6/2/2010)

GS-SDS-Oral 2010

page 73

Surgery Review Course

page 74

GS-SDS-Oral 2010

GS_SDS_Oralrevised03012010 (6/2/2010)

The Osler Institute

Gastrointestinal Bleeding
Charles D. Goldman, MD Des Moines University/Mercy Medical Center Des Moines

I. Esophageal sources
Although can get significant bleeding from Barretts ulcers, only question asked is going to be acutely bleeding varices. Initial attempt should be made to control endoscopically either with sclerotherapy or rubber band ligation. Some evidence latter is more effective. If this fails or only partially controls hemorrhage, medical therapy is indicated. High dose continuous infusion octreotide is now favored over use of vasopressin, due to at least equal, if not superior, efficacy without attendant cardiac side effects. If patient still bleeding, most would now go to TIPS procedure rather than balloon tamponade. TIPS is especially indicated if patient is candidate for transplant and therefore cannot undergo a surgical shunt procedure. In the rare case of failure of all the above, emergency portacaval shunt (not the Warren shunt as it takes too long to decompress the portal system and is too lengthy an operation for these usually sick patients) or stapled esophageal transection should be performed.

A. Gastric sources
Mallory-Weiss tear: Follows forceful emesis, often with full stomach with pre-existent Type I hiatal hernia. Risk population is alcohol abusers and pregnant women. Attempt to control endoscopically first. Some report use of nasogastric decompression and acid suppression after endoscopy, but the value of these is uncertain. If still bleeding, can try selective vasopressin if no cardiac contraindication, although more likely now to try to embolize. Final recourse is gastrotomy with oversewing of the tear. Bleeding gastric ulcer: rarely will cease bleeding with endoscopic measures. Key is to avoid rebleeding as this is usually the mortal event in these oftentimes-sick patients. Make initial endoscopic attempt anyway, be sure to biopsy to rule out malignancy. If conservative measures fail, definitive OR in good risk patients (antrectomy with a vagotomy added for type II and III ulcers only) while should excise ulcer +/- vagotomy and pyloroplasty (availability of PPIs make the need for this marginally helpful anti-ulcer operation less certain) in the unstable patient Stress gastritis: antral sparing is characteristic of this form of stress-induced ulceration. Thought to be combination of presence of acid in a hypoperfused stomach in critically ill patient. Most ICU patients can be
GS_SDS_Oralrevised03012010 (6/2/2010)

GS-SDS-Oral 2010

page 75

Surgery Review Course


demonstrated to have these changes, but with use of pH-monitored acid reduction and/or cryoprotectants such as Carafate profuse bleeding is now rare. If preventive measures fail, reduce gastric perfusion further with octreotide or vasopressin. If these measures are ineffective and patient has significant transfusion requirement, take to OR. If good risk patient, operation with lowest rebleed rate is near-total gastrectomy, leaving just enough stomach to use for anastomosis of gastric remnant to Roux limb. If patient too unstable for this, gastric devascularization is the best compromise between rebleed rates and operative mortality. Ligate right and left gastric arteries and both gastroepiploic vessels (remember that left gastroepiploic arises from splenic artery. Need intact short gastric system to offer devascularization as unlikely stomach can survive solely on trans-esophageal and transpyloric collaterals. Some literature on doing gastric devascularization as percutaneous embolization procedure in IR. Duodenal ulceration: Endoscopic measures first. Choice of operation is based on a number of factors: can patient tolerate stress of rebleed? (If not, lean toward more definitive operation). Bleed NSAID-related? (If so, ligation of bleeder with no antiulcer operation might be best). What is H. pylori status? (If dont know, lean toward lesser operation, perhaps parietal cell vagotomy). Can patient tolerate longer operation? (HSV takes the longest, V and P the shortest). Does patient have long history of severe ulcer disease? (If so, lean toward vagotomy and antrectomy because of lowest rate of recurrent ulceration). Even if you open the pylorus to oversew the bleeder, you can reclose ends of pyloric muscle and still do HSV. Alternative is to simply open the duodenum without traversing the pylorus to oversew the ulcer.

B. Lower GI sources
When presented with massive lower GI bleed, first test is rigid proctosigmoidoscopy. If negative, go to arteriography next. While labeled RBC scan is more sensitive to slower rate of bleeding, timing of scanning is critical and often leads to erroneous localization of the source of bleed. If bleeding site is identified selective vasopressin infusion has generally been used in the past, while there are more recent reports of selective embolization of bleeders with Gelfoam and coils, especially in the more thick-walled left colon. Obviously, this carries some risk of bowel ischemia (especially in the right colon) so patient needs to be monitored closely. If nonoperative measures fail and arteriography has defined
page 76 GS-SDS-Oral 2010
GS_SDS_Oralrevised03012010 (6/2/2010)

The Osler Institute


a site of bleeding, segmental colonic resection is appropriate. If arteriography is negative, this implies that bleeding is now subacute and colonoscopy is the next step. Problematic patient is the one in whom the bleeding stops so all studies are negative but they continue to have a slow but persistent GI bleed. Usually end up with repetitive nuclear scans and arteriography without localization. If colonoscopy does not make diagnosis, take to OR for push enteroscope. These patients are often found to have multiple gastric or small bowel AVMs. While cauterization of the more prominent may be helpful, another alternative is treatment with low-dose birth control pills (men also). Just published study in Am J Gastroenterol found almost 100% control of bleeding as long as therapy was continued, while 12 of 14 patients who stopped meds rebled. Mechanism of action uncertain. In patient who is bleeding out from colon and cannot be transported for arteriography, subtotal colectomy with ileorectal anastomosis is the most conservative alternative. In the elderly patient, creation of an ileorectostomy is associated with a high incidence of diarrhea and incontinence. In the patient who has no evidence of significant left-sided diverticular disease, bleeding source is almost invariably an AVM of the right colon. Reports of use extended right hemicolectomy in preference to a total abdominal colectomy has shown equivalent control of bleeding and should always be considered as an option in the elderly patient to leave more fecal reservoir in these patients with marginal anal continence.

GS_SDS_Oralrevised03012010 (6/2/2010)

GS-SDS-Oral 2010

page 77

Surgery Review Course

page 78

GS-SDS-Oral 2010

GS_SDS_Oralrevised03012010 (6/2/2010)

The Osler Institute

Hepatobiliary Oral Board Questions


Charles D. Goldman, MD West Virginia University

Liver and Gallbladder I. Bile Duct Injuries


Likely to be following laparoscopic cholecystectomy. Confirm injury or leak with ultrasound guided fluid aspiration and/or HIDA. Define anatomy with ERCP and transhepatic cholangiography. Drain the biloma percutaneously. If site of leak is cystic duct stump, most will stop within a week. Can hasten process by placing stent across the leak endoscopically. In the case of injuries to the common hepatic or common bile duct: Unless there is a transection or clipping which involves < 50% circumference of the duct, repair should be hepaticojejunostomy utilizing a Roux loop. For the previously mentioned lesser injuries, can attempt primary repair +/- T-tube (in small ducts, trend is to not employ a Ttube as it leads to higher rate of stenosis).

II. Cholangitis
Aim of therapy is to decompress duct after fluid resuscitation and antibiotics are begun. Clearing the duct of stones is a secondary consideration, especially in the patient who is very sick. Get ultrasound first to get some idea of level of bile duct obstruction since if cause of obstruction is lesion in intrahepatic ducts transhepatic approach is preferable to endoscopic. Given present interventional capabilities, order of therapies is usually endoscopic attempt to dislodge the stones (if they are the cause benign biliary stricture is the second most common etiology), cut a sphincterotomy and stent the duct open. If this is unsuccessful, transhepatic decompression is performed. If this too fails, open patient to decompress the duct. If patient highly unstable, place T-tube and get out. If more stable, try to get stone out. If stone impacted, consider either a duodenal approach to the stone with open sphincterotomy or do biliary drainage procedure.

III. Choledochal Cysts


Only one asked on the Boards is the extrahepatic cyst of the common bile duct, the Type I. Patients with choledochal cysts get biliary stasis yielding choledocholithiasis, pancreatitis and an increased risk of bile duct malignancies. Although optimally the treatment is complete excision with hepatico jejunostomy, this is rarely possible due to inflammation around the cyst. Alternatively an intramural dissection of the inner wall of the cyst is performed, the distal
GS_SDS_Oralrevised03012010 (6/2/2010)

GS-SDS-Oral 2010

page 79

Surgery Review Course


common bile duct is ligated, and a cystojejunostomy is created.

IV. Acalculous Cholecystitis


Probably the most overdiagnosed entity in the sick patient in your ICU. Associated with lack of bowel function and TPN yielding stasis of bile in non-contractile gallbladder. Ultrasound should show some wall thickening, while fluid around the gallbladder is non-specific. HIDA scan may be helpful if serum bilirubin is not too high to allow uptake of tracer; doing HIDA in conjunction with IV morphine to raise common bile duct pressure felt to enhance the predictive value of the positive HIDA (no filling of GB) in this disease entity. Treatment is cholecystectomy if patient is stable enough, percutaneous drainage if they are not (which is usually the case).

V. Bile Duct Malignancies


Cholangiocarcinoma tends to be a slow-growing tumor so chemotherapy has no role and radiation therapy has limited activity. Lesions of the distal common bile duct can be addressed with either Whipple procedure or resection of the suprapancreatic CBD with clearance of the lymphatic tissue in the hepatic triad and along the hepatic artery. These procedures are appropriate to perform even in the presence of limited metastatic disease since they palliate the patients biliary obstruction. For tumors at the junction of the hepatic ducts (Klatskin lesion), most are unresectable at diagnosis. If so, decompress biliary system by Roux-en-Y to extrahepatic portion of left main hepatic duct that is approached by taking down the hilar plate at the base of the umbilical fissure. If this area is also involved, other alternative drainage procedures are placement of a percutaneous U-tube or incision into segment III to expose a more distal branch of the left hepatic duct. If the cholangiocarcinoma is intrahepatic, it may be amenable to liver resection for cure. For gallbladder lesions, cholecystectomy is adequate for cancers involving the mucosa only and possibly for nontransmural lesions on the non hepatic aspect of the gallbladder. For deeper lesions with no distant mets or nonmucosal lesions on hepatic aspect of gallbladder, traditional answer is to perform wedge resection of gallbladder bed and portal lymphadenectomy. Japanese routinely add excision of the extrahepatic common duct with hepaticojejunostomy to make lymph node clearance more complete. Problem is that if lymph nodes are positive there are few five-year survivors (although as with gastric cancer, the Japanese experience differs from the American in claiming significantly increased survival with aggressive extirpative surgery) which makes the extended resection for stage II lesions more of a staging procedure than a therapeutic one. Radical cholecystectomy and even Whipple operation has been described, again
page 80 GS-SDS-Oral 2010
GS_SDS_Oralrevised03012010 (6/2/2010)

The Osler Institute


especially by the Japanese, for stage III lesions but their therapeutic value is based on anecdote not series with adequate statistical power.

VI. Liver Cysts


Cysts that appear simple on CT scan can generally be left alone, no matter what the size. If it should become symptomatic for example, pain aspirate the cyst. If no bile or pus in aspirate, ablate with alcohol percutaneously. If pus, drain until cessation of output. If bile in aspirate, define communication with bile duct either with cystogram or ERCP. Do not inject alcohol as it acts as a sclerosant in the biliary system. Remove drain if becomes symptomatic once again, unroof the cyst, ligate biliary communication, and fill the defect with omentum. Remove gallbladder to avoid confusion if there is future complaint of right upper quadrant pain. Hydatid cysts have characteristic CT appearance calcified walls with daughter cysts. If suspect echinococcal disease, get serology. If positive, start on medical therapy but will still need to deal with hepatic lesion. As with simple cyst, due to use of sclerosing agent (hypertonic saline or formalin) need to be sure there is no biliary communication do not aspirate any cyst you suspect to be echinococcal as spillage can lead to anaphylaxis, so ERCP is the test of choice. Due to inflammatory response around cyst, cyst excision is often difficult and bloody. Most prefer pericystectomy or, at the least, unroofing of the cysts. Since both of these violate the cyst, prior to any incision of the cyst, aspirate the cyst and sclerose with either hypertonic saline or formalin.

VII. Liver Neoplasms


Look at clinical history first to formulate likely diagnosis for liver nodules. If patient is young female on birth control pills, hepatic adenoma is far more common than hepatoma. If patient has chronic hepatitis (either B or C) or alcoholic cirrhosis, think hepatoma. Prior history of cancer or biopsies in past, even if read as benign, consider metastatic disease. CT is generally first test. Helical CT with rapid contrast infusion can often diagnose hemangioma, although MRI or tagged red cell scan are generally obtained to confirm diagnosis. CT alone can differentiate focal nodular hyperplasia and adenoma. If scans suggest hepatoma, do not percutaneously biopsy if considering resection. There is some evidence for seeding of biopsy tract with hepatoma. Get serum AFP; if elevated and clinical scenario is appropriate, take patient for resection. No biopsy is needed for hemangioma either reliably diagnosed by scans and the very rare chance this is hemangiosarcoma cannot be ruled out by percutaneous biopsy anyway. Percutaneous core may be able to differentiate between FNH and adenoma.
GS_SDS_Oralrevised03012010 (6/2/2010)

GS-SDS-Oral 2010

page 81

Surgery Review Course


Hemangiomata do not need to be resected unless they are causing biliary tract compression, pain, or thrombocytopenia due to platelet trapping. Presence of liver hemangioma is not contraindication to future pregnancy. FNH can be observed with serial scans to assure stability. Hepatic adenoma has a high rate of spontaneous bleeding so if regression does not occur with cessation of BCPs, they need to be resected. In considering resection for hepatoma, must rule out extrahepatic disease that contraindicates resection since there is no palliative value to hepatectomy (think Mickey Mantle) and whether there is adequate liver reserve to allow lobectomy or even trisegmentectomy (the reason 30-day mortality after resection is so high in cirrhotics). If your assessment is that the patient will not have adequate functioning liver remaining after resection (and there is no good test for that although various polysaccharide clearance tests have been put forth their predictive value is uncertain. This assessment is a mix of eyeballing the patient and the usual Childs criteria), might consider hepatectomy with transplant. Alternative to resection is percutaneous cannulation of the hepatic artery and selective chemoembolization of the arterial inflow of the tumor. Some surgeons have combined this with portal vein embolization; this leads to hypertrophy of the non-embolized side and may increase the amount of hepatic tissue such that inoperable patients can be rendered candidates for resection. Factors favoring use of transplant are younger age of patient and fibrolamellar pathology. Palliative measures for unresectable hepatoma include chemoembolization, alcohol ablation, possibly radiofrequency ablation, and external beam RT +/- adriamycin.

page 82

GS-SDS-Oral 2010

GS_SDS_Oralrevised03012010 (6/2/2010)

The Osler Institute

Pancreas
Charles D. Goldman, MD Des Moines University/Mercy Medical Center Des Moines

I. Pancreatitis
Little new to discuss concerning the role of surgery in acute pancreatitis. There is renewed interest in the use of peritoneal lavage in cases of the severely ill (by Ranson criteria) and some reduction in 30-day mortality has been seen when lavage was used from day 2-14. Debridement of the retroperitoneum in fulminant pancreatitis remains a matter of debate. All agree that if retroperitoneal aspiration shows organisms on Gram stain and the patient is continuing to deteriorate, exploration is warranted. The evacuation of sterile necrosis is not as avidly pursued in the US vs Europe, as American data does not support its efficacy. At operation, the non-cohesive cottage cheese is scooped out. If it can be located, the splenic artery should be ligated to try to reduce the risk of its erosion and hemorrhage. If drains are used, they should be sizable, sliding Penrose drains over them to protect them from becoming clogged with debris. Alternatively, any of a number of open abdomen techniques can be employed. Feeding jejunostomy should be placed. Primaxin is generally antibiotic of choice, with consideration to fungal coverage in the patient who has been on a prolonged course of antibiotics Pseudocysts, particularly communicating ones, are more likely with chronic pancreatitis. There has been a drift toward more conservative management of pseudocysts, observing those up to 5 cm that are causing no symptoms. For those that are larger or are symptomatic (pain, early satiety, previously treated abscess, bleeding into the pseudocyst) surgical decompression is recommended. While endoscopic transgastric techniques have gained some adherents, open cystgastrotomy or Roux-en-Y pancreaticojejunostomy (erroneously referred to as the Puestow) are the most common ways in which pseudocysts are managed. The most debilitating long-term consequence of chronic pancreatitis is pain. A CT must be obtained first to make sure a pseudocyst is not the underlying problem. If no pseudocyst is found, ERCP or MRCP to define the ductal pathology is mandatory. A dilated duct, whether chain-of-lakes or enlarged behind a single obstruction, is treated with side-toside pancreaticojejunostomy. Small duct disease cannot be drained, so gland must be resected. The suggested technique is the Frye procedure in addition to opening up the dilated pancreatic duct to the left of the mesenteric vessels, the head

GS_SDS_Oralrevised03012010 (6/2/2010)

GS-SDS-Oral 2010

page 83

Surgery Review Course


of the pancreas is cored out and then drained into a long side-to-side pancreaticojejunostomy. An unusual consequence of chronic pancreatitis is pancreatic ascites. Often attributed to the rupture of a communicating pseudocyst, the patient presents with anorexia, mild abdominal pain and profound abdominal distention. Although CT is often listed as the first test to get, it is generally unhelpful. Tapping the ascites first will characterize the source of the problem. Needless to say, pancreatic ascites is astronomically high in amylase and protein. Once the diagnosis is made, the patient is made NPO, placed on TPN, and started on octreotide. Repeated abdominal drainages are performed; if pleural effusions develop, they should be drained with chest tubes. After the initial phase of stabilization, an ERCP is obtained to hopefully define the site of the ductal disruption. Placement of a stent across the leak, if possible, or simply sphincterotomy to decompress the pancreatic duct may hasten healing of the leak. Failure of endoscopic attempts to seal the leak necessitate drainage of the leak into a Roux limb.

II. Pancreas Divisum


Non-fusion of dorsal duct with ventral duct, leaving the majority of the gland drained via the smaller dorsal duct that terminates at the minor papilla. Thought to cause either recurrent pain episodes with transient hyperamylasemia or chronic pancreatitis. To confirm ductal anatomy as causative factor in pain, observe size of duct and patient symptoms after IV bolus of secretin. ERCP should follow to define anatomy and possibly perform dorsal sphincterotomy +/- stent placement to see whether pain or pancreatitis episodes abate. If so, should episodes recur after stent removal, open dorsal sphincterotomy would be indicated. If treatment of sphincter does not decrease symptoms/attacks, other options are Puestow for those with a chronically dilated duct and distal pancreatectomy for those with non-enlarged pancreatic duct. Both of these procedures assume some underlying chronic pancreatitis and glandular fibrosis, to allow for creation of the anastomosis in the case of the Puestow and to delineate what portion of the pancreas should be resected in the latter procedure.

III. Pancreatic Cancer


Given classic presentation of patient with weight loss and painless jaundice, initial work-up is usually CT. Helical three-phase CT has sufficient definition to assess vessel encasement so angiogram is generally not needed. Look for non-regional adenopathy, liver metastases, peritoneal studding on CT as these would contraindicate an attempt at Whipple procedure. Even if patient is jaundiced, unless they have coagulation abnormalities, do not initially decompress the bile duct since this increases operative wound
page 84 GS-SDS-Oral 2010
GS_SDS_Oralrevised03012010 (6/2/2010)

The Osler Institute


complications. Routinely obtain endoscopic ultrasound to further assess for adenopathy and vessel involvement, as well as obtaining tissue diagnosis. What to do next is controversial. If at MGH, you take the patient for laparoscopy and peritoneal washings. If non-regional disease is encountered or washings are positive for malignant cells, palliation should be done without operation if possible. At MD Anderson, you would be staged with laparoscopy and if found to have no non-regional disease, preoperative chemoradiation would be given. Others would just take patient to OR to assess resectability feeling that a Whipple operation is the best palliation even in the presence of incurable disease. If you opt for the latter, make reasonable attempt to get an intraoperative pathologic diagnosis, either by shave biopsy of the suspicious lesion or by transduodenal FNA/CNB, if you do not have preoperative tissue diagnosis. If none of these are diagnostic but the clinical scenario suggests cancer (e.g., painless jaundice in a non-drinker whose pancreas shows no inflammatory changes to suggest pancreatitis as the cause of the biliary obstruction), proceed with the Whipple even if no diagnosis biopsy. Need to resect and repair or replace vein is not a contraindication to resection, although these borderline cancers should likely be offered preoperative chemoradiation and restaging before attempting pancreaticoduodenectomy. NCCN standards of unresectability are: distant metastases, greater than 180 degree encasement of superior mesenteric artery, any celiac abutment, unreconstructible SMV/portal occlusion, or aortic invasion/encasement. Despite folk-lore to the contrary, two large reviews have shown that biliary bypass can be done either to the gallbladder (as long as you can demonstrate cystic duct patency) or the common bile duct with equivalent success. However almost all American surgeons use the common duct for reconstruction. Recent prospective trial out of Johns Hopkins claims that all patients should receive a prophylactic gastrojejunostomy, so probably safe to give this as Board answer if patient is open and only needs palliation. Unfortunately, study was done prior to the introduction of the chemotherapy agent gemcitabine that has shown modest prolongation of survival but impressive enhancement of quality of life, including reduction in incidence of gastric outlet obstruction by tumor. Further, many of the pancreatic cancer patients who have recurrent emesis will not be helped by diversion; their emesis is due to retroperitoneal nerve invasion by the tumor. Pyloric sparing Whipple is an appropriate oncologic choice versus standard antrectomy, but is associated with more gastric retention in the immediate postoperative period. Whether all resected pancreatic cancer patients should receive postoperative adjuvant therapy is uncertain. If they are

GS_SDS_Oralrevised03012010 (6/2/2010)

GS-SDS-Oral 2010

page 85

Surgery Review Course


offered therapy, first line option is gemcitabine with or without 5-FU/radiation.

IV. IPMN (Intraductal papillary mucinous Neoplasms)


Widespread use of CT scanning led to realization that up to 25% of pancreata contain small mucinous lesions of variable neoplastic potential. IPMNs are lesions connected to the ductal system that elaborate mucin. M:F ratio is 1:1 (compared to 1:20 with mucinous cystic neoplasms), age of discovery is around 65 years, 2/3 are in the head (<90 of MCNs are in the body/tail). 70% involve the main duct. Main duct IPMN is characterized by ductal dilatation of >1 cm diameter with associated nodularity. MRCP is seemingly more sensitive than CT in evaluating these lesions, as is endoscopic ultrasound. Can aspirate these lesions under ultrasound to check cytology and intra-lesional CEA levels. Main duct IPMNs have a high risk of subsequent malignancy so surgical resection is recommended for all who are reasonable surgical candidates. Resection is partial pancreatectomy guided by intraoperative ultrasound assessment of the extent of tumor; although some believe there is a field effect to main duct IPMN and total pancreatectomy should be done, recurrence rates are similar for partial vs. total pancreatectomy. As branch-duct IPMNs have a much lower rate of malignancy, aselective approach is employed in their case. Simple cysts that are <3 cm in diameter and have no mural nodules can be observed with periodic MRCP surveillance. If they need to be resected, segmental resection should be done due to the low rate of associated malignancy.

page 86

GS-SDS-Oral 2010

GS_SDS_Oralrevised03012010 (6/2/2010)

The Osler Institute

Spleen
Charles D. Goldman, MD Des Moines University/Mercy Medical Center Des Moines

Questions for Oral Boards: Spleen


There are relatively few topics that can be asked on the orals regarding the spleen. The most common would relate to splenic injury in trauma. While splenic salvage is desirable, the incidence of postsplenectomy sepsis in the adult trauma patient is very low, so if there are multiple associated injuries and/or the patient is unstable, just take the spleen out. There is little evidence that reimplantation of small segments of splenic tissue into the omentum restores any of the immunologic function of the spleen, and the risk of splenosis probably exceeds any theoretical benefit, so again would not pursue. If can do less than total splenectomy and maintain inflow/outflow from splenic system (short gastric flow alone is insufficient to support the red cell and antigen clearing functions of the spleen), conservative operation is worth pursuing. Be aware of various techniques including use of argon beam coagulator, platelet/fibrin glue for broad surface bleeding, oversewing with/without pledgets, compression wrap with Vicryl mesh. In the pediatric patient (and some adults) splenic conservation is more feasible. Use CT scan to characterize extent of injury and assess amount of blood in abdomen. If only one pole of spleen appears to be involved, can consider percutaneous embolization of that subsegmental branch of the splenic artery. Even in the pediatric population where postsplenectomy sepsis is more common, tendency is to not transfuse more than 2 units of blood to stabilize patient. If it appears more blood replacement will be necessary or the child becomes hemodynamically unstable, operate. Even in children, postsplenectomy sepsis is rare among trauma victims; most of the cases have followed surgery for hematologic disorders, usually congenital hemolytic anemias such as spherocytosis. Another favorite topic is ITP, immune (or idiopathic) thrombocytopenic purpura. Presentation is usually a postadolescent female with the onset of easy bruising, bleeding from gums with trivial trauma, or menorrhagia. In children, ITP generally follows an acute viral infection, often in the springtime, and is self-limited in over 85% of pediatric patients. Antecedent infections rarely noted in the adult setting. There is no splenomegaly (except in the cases that are associated with a preceding infection or hematologic malignancies) since this is not a sequestration disease, but rather is caused by destruction of IgG coated platelets in the spleen. The blood smear may show immature platelets; bone
GS_SDS_Oralrevised03012010 (6/2/2010)

GS-SDS-Oral 2010

page 87

Surgery Review Course


marrow biopsy is mandatory to make diagnosis of ITP as this is a diagnosis of exclusion. If patient has depressed white or red cell counts in addition to thrombocytopenia, this points to a more global bone marrow process and should not be diagnosed as having ITP. Also rule out collagen vascular disease and assess for signs of lymphoma/leukemia. ITP patient will have normal to elevated numbers of megakaryocytes seen on bone marrow aspirate. Measurement of serum anti-platelet antibodies not helpful as levels of antibody do not correlate with severity of disease. Initial treatment is immunosuppression; steroids alone (prednisone 1 mg/kg) if the patient is not having serious hemorrhage, steroids plus anti-IgG globulin if there is significant hemorrhage. Although they have more serious side effects, may have to employ TPO agonists if not responding. Since adult form of disease tends to become chronic in the adult, aim is to wean patient off steroids and see if adequate platelet counts can be maintained. If fails steroids, can try Danazol (although in this predominantly female disease, the androgenic side-effects of Danazol often lead to its discontinuation). Laparoscopic splenectomy is indicated if patient cannot be maintained off immunosuppressives since long-term steroid use is deleterious in young patients. 60-75% of ITP will remit with splenectomy. If patient continues thrombocytopenic after splenectomy, look at blood smear. If greater than 2% of cells have Howell-Jolly bodies (nuclear debris in red cells that is usually cleared in the spleen), patient is asplenic and no sense looking for accessory spleens. In that case, patient is placed on lifetime immunosuppression with agents such as AZT, cyclosporine or cytoxan if platelet counts are less than 50K. If Howell-Jolly bodies are absent, get CAT scan or damaged red cell scan to look for accessory spleen. Even if found, removal of accessory spleen only corrects thrombocytopenia in ~60% of patients

page 88

GS-SDS-Oral 2010

GS_SDS_Oralrevised03012010 (6/2/2010)

The Osler Institute Case 1


A 65-year-old male comes to you complaining of a two-week history of increasing difficulty swallowing solid food. He is not vomiting. It just seems the "food is sticking behind his breastbone." He has lost about ten pounds in the last month, although his appetite is good. He has a 60-pack year history of smoking and drinks a fair amount of beer on the weekends. 1. What is your differential? Answer: Esophageal cancer, stricture from reflux, diverticula, and motility disorder
2. What is your initial workup? Answer: R/O obvious mets from cancer on physical exam\CXR\LFTS; UGI and EGD to confirm presence of cancer 3. The biopsy shows a circumferential squamous cell carcinoma at 35 cm, nearly obstructing. What next? Answer: (You must start from worst case scenario, that being patient is not an operative candidate due to distant mets or locally advanced disease.) R/O distant mets with chest/abdominal CT; mediastinoscopy if CT is suspicious for upper mediastinal adenopathy. Assess depth invasion with CT, possible endoluminal ultrasound. Get bronchoscopy if CT or symptoms suggest TE fistula, a definite sign of inoperability. (No mets.) Question enlargement of lymph nodes at esophageal hiatus.

4.

What are treatment options? Answer: Chemo rads as only therapy vs esophagectomy with/without preoperative chemo/RT/chemo-RT. 5. Decide on OR, what preop tests do you want? Answer: PFTs mandatory as aid in deciding whether thoracotomy is safe. Might want to workup heart due to smoking history. Discuss indications and conduct of OR: transhiatal esophagectomy vs Ivor-Lewis approach.

Case 2
A 45-year-old male comes into the ER complaining of painless abdominal distension about 3 weeks after recent hospitalization for acute exacerbation of his alcohol-induced chronic pancreatitis. Exam reveals a massively protuberant abdomen, dull to percussion, with positive fluid wave, also has notable peripheral muscle wasting. 1. What do you do to make the diagnosis? Answer: CT or U/S of abdomen to check pancreas and to confirm ascites; tap abdomen and send fluid for usual labs; get routine bloods and X-rays, including CXR to R/O pleural effusion. (The peritoneal fluid has an amylase of 10,000 and a high protein level; serum amylase is 80.) 2. What is your diagnosis? Answer: (pancreatic ascites)

3. What is the cause of the ascites?


GS_SDS_Oralrevised03012010 (6/2/2010)

GS-SDS-Oral 2010

page 89

Surgery Review Course


Answer: (In alcoholic pancreatitis, almost always rupture of the pancreatic duct; less likely, pseudocyst rupture) 4. Treatment? Answer: Repeated abdominal taps; thoracentesis if symptomatic; support nutrition with TPN (feeding distal to pancreas probably not good answer); somatostatin to dry up secretions. (After 7-10 days, still requiring daily paracenteses). 5. What do you want to do?
Answer: Get ERCP and see if a leak can be identified and place stent across. (ERCP shows ductal leak, but cannot stent across.)

6. What next? Answer: Continue conservative RX for 3-4 weeks and if still leaking, take to OR to drain leak into longitudinal pancreaticojejunostomy. 7. Point: this is not a surgical emergency and these patients are very protein depleted. Be sure to adequately aliment these patients before you explore.

Case 3
A 74-year-old male comes to you complaining of gurgling in his neck, regurgitation of undigested food, and bad breath. Appetite OK; no weight loss. Physical exam unremarkable. What workup do you want? Answer: Need to R/O esophageal motility disorder vs diverticulum. Get UGI first, not endoscopy as latter runs the risk of perforation with Zenker's. (UGI reveals a Zenker's diverticulum.) Ask what size, as this can affect your choice of operative therapy. How do you repair this? Answer: If less than 2 cm, can simply do myotomy of cricopharyngeal muscle; if larger, do myotomy and either diverticulectomy or suspend diverticulum from precervical fascia. What side of the neck do you make incision? Answer: Generally on left, paralleling sternocleidomastoids anterior border. After the operation, the patient is notably hoarse with slight inspiratory stridor. What's the problem? Answer: Recurrent injured laryngeal nerve, probably from retraction. Most will come back to normal; if not, be prepared to discuss options for treatment of vocal cord paralysis.

1.

2.

3.

4.

page 90

GS-SDS-Oral 2010

GS_SDS_Oralrevised03012010 (6/2/2010)

The Osler Institute Case 4


A 45-year-old female comes to your office complaining of redness of her left breast. She is sexually active. No history of trauma to the breast. What is your differential? Answer: Mastitis vs inflammatory breast cancer vs superficial thrombophlebitis Treatment? Answer: Okay to try short course of antibiotics with follow-up within week. If no improvement (there won't be), incisional biopsy of area through both reddened area and normal adjacent skin. Biopsy shows inflammatory breast cancer. What is the diagnostic pathologic finding? Answer: (Tumor cells in subdermal lymphatics) Treatment? Answer: Local therapy has little influence on outcome; chemotherapy, either multi-agent or Taxol alone, instituted and maintained 2-3 cycles past clinical complete response; mastectomy after initial chemo course augments local control and allows one to assess for complete pathological response. If complete response confirmed by path specimen, D/C chemotherapy and radiate chest wall to decrease risk of local recurrence; place patient on ovarian ablation agents such as Zoladex or Tamoxifen.

1.

2.

3.

4.

Case 5
A 85-year-old patient with organic brain syndrome is brought to your ER from a skilled care facility due to abdominal distension and no bowel movement for 24 hours. She had been having trouble with occasional emesis for the last 6 days. Your initial assessment? Answer: Confirm tympanitic distension, not ascites; check for previous abdominal incisions; check hernial orifices and do rectal. What labs do you want? Answer: Usual blood work; obstructive series. (WBC is 16,000; X-rays show SBO with no air in rectum. Psoas border is obscured in RUQ and there is air in the gallbladder fossa.) Patient has gallstone ileus with history of "tumbling obstruction." What do you want to do next? Answer: Place NG to decompress; take to OR as soon as patient is adequately fluid resuscitated. How do you perform the operation (the critical part of question)? Answer: Make enterotomy proximal to impacted stone (not directly over it) and milk stone back to enterotomy. Run rest of proximal bowel to be sure there is not another stone. If patient
GS-SDS-Oral 2010 page 91

1.

2.

3.

4.

GS_SDS_Oralrevised03012010 (6/2/2010)

Surgery Review Course


is stable, can definitely deal with cholecystenteric fistula by performing cholecystectomy and closure of enterotomy. Otherwise, do cholecystostomy and evacuate remaining stones and deal with GB electively at much later date.

Case 6
A 21-year-old fraternity member comes to your ER complaining of severe abdominal pain with vomiting. He has been drinking beer for 36 hours, since the end of finals. 1. What is your initial differential? Answer: Gastritis, pancreatitis, perforated ulcer, spontaneous rupture of esophagus given history of emesis. 2. Important points on physical exam? Answer: Feel for crepitus over chest and neck; look for signs of hemorrhagic pancreatitis. What labs and X-rays do you want? Upright CXR to R/O pneumothorax as sign of Boerhaave's syndrome and to R/O pneumoperitoneum; obstructive series; usual bloods, including amylase and lipase. (CXR WNL; abdomen shows isolated loop of dilated small bowel; amylase 3400, total bilirubin 1.5.) Probably pancreatitis. NG, IVFs, analgesics, close monitoring; some might get U/S to R/O gallstones despite history EtOH, as binge drinking is much less common cause of pancreatitis than chronic EtOH. Some might get CT for baseline exam. If feel attack is severe, might start Primaxin per recent article in GI literature; somatostatin unproven in this setting. (Patient improves to point that able to feed on day 10, but soon after resuming PO, he starts vomiting and you can appreciate an epigastric mass.) CT scan or ultrasound to confirm pseudocyst? 3. How do you manage this? Answer: Do not externally drain unless evidence of infection; most of these will resolve without need for aggressive treatment; feed with enteral tube past ampulla or use TPN; reassess patient in 6-8 weeks to see if still symptomatic and if pseudocyst is resolving; might get ERCP, as communication with duct, is predictor of resolution. 4. What operations can you do to drain this pseudocyst internally, if it persists? What is the advantage of waiting 6-8 weeks after discovery of the pseudocyst to consider operative drainage? 5. What are some long-term complications of an undrained pseudocyst? Answer: Pain, biliary/duodenal obstruction, infection, hemorrhage into cyst either from erosion into stomach or splenic vein, splenic vein thrombosis with development of gastric hypertension.

page 92

GS-SDS-Oral 2010

GS_SDS_Oralrevised03012010 (6/2/2010)

The Osler Institute Case 7


A 64-year-old female comes to your office complaining of 3-4 months of feeling depressed. Recently, has been fatigued, and has poor appetite with a ten-pound weight loss over last 3 weeks. On exam, the patient's sclerae are icteric and she has a RUQ mass. What is the RUQ mass? Answer: Most likely, dilated liver. The classic finding of dilated gallbladder is only one-fourth as common as an enlarged liver. What tests would you like? Answer: Liver functions, PT/PTT, CEA, CA 19-9. You obtain a CT which shows a mass in the head of the pancreas. What next? Answer: May be tempted to do percutaneous FNA, but if tumor seems resectable on CT, not worth the risk of disseminating tumor. Get ERCP for ampullary cytology, to determine if this is a more favorable ampullary or duodenal carcinoma and to R/O common duct stone as cause of jaundice. Only stent the bile duct if jaundice is severe with unrelenting itching or deranged liver functions, especially coags. Stenting raised rate of infectious complications if Whipple attempted. Biliary cytology is negative and ERCP shows double duct sign consistent with neoplasm. Ask for angio with venous phase to R/O replaced right hepatic artery and to assess portal vein involvement. Angiogram shows tumor not encroaching on portal vein. Take to OR. You will be asked how you Whipple procedure. Do you biopsy the mass? Answer: (I do not.) What are your indications for doing the Whipple once you explore the patient? Answer: (Decide this for yourself before you take the orals.) Some key points: laparoscope patient before you open. It is more sensitive for peritoneal implants than CT. If you find these, can do your biliary and gastric bypass laparoscopically. Lavage the abdomen with one liter of saline on entry and send for cytology. If this is positive for malignancy, outcome is poor even in resectable patient and this might dissuade you from doing Whipple for marginal indications. No apparent difference in oncologic outcome with standard Whipple vs pylorussparing; give patient jejunostomy catheter for immediate postop feeding.

1.

2.

3.

4.

5.

GS_SDS_Oralrevised03012010 (6/2/2010)

GS-SDS-Oral 2010

page 93

Surgery Review Course Case 8


A 44-year-old IBM executive comes to see you for unrelenting abdominal pain related to alcohol-induced chronic pancreatitis. He requires Fentanyl patch to control his pain and takes 30 Pancrease daily. He has not had a drink in over 10 years, but had to take leave of absence from job due to worsening pain. He is not diabetic. Which patients would you consider for surgical palliation of chronic pancreatitis? Answer: Poorly compliant, socially disadvantaged, and most especially, continued substance abusers are poor choices. What test is critical in treating this patient? Answer: ERCP delineates ductal causes of pain. Dilated duct with single stricture (uncommon) vs "chain-of-lakes" (very common), or normal size duct which implies parenchymal disease as cause; also, get ultrasound or CT to be sure that pseudocyst is not present. What are some complications of chronic pancreatitis other than pain, diabetes, and malabsorption? Answer: Pseudocyst, biliary obstruction, splenic vein thrombosis If the patient is a suitable operative candidate and has a "chain of lakes" ductal anatomy, what procedure is appropriate? Answer: Longitudinal pancreaticojejunostomy, the incorrectly named Puestow procedure. Know how to describe. In particular, you do not anastomose mucosa of intestine to mucosa of duct. You need to extract pancreatic stones and duct should be opened well into head of pancreas. If the patient has a normal duct, what can you offer? Answer: Total pancreatectomy vs Whipple vs central pancreatic transection vs Fry procedure. Know how to generally describe and also be aware of complications, especially brittle diabetes with total resection.

1.

2.

3.

4.

5.

Case 9
A 30-year-old female that you performed an apparently flawless laparoscopic cholecystectomy on for asymptomatic cholelithiasis returns to your office 10 days later with obvious jaundice and RUQ pain. 1. What is your differential? Answer: Bile leak, iatrogenic occlusion of common duct, postop pancreatitis 2. Where could the leak be coming from? Answer: Clip fell off of cystic duct, partial transection of common bile duct, leakage from accessory bile ducts in gallbladder bed
page 94 GS-SDS-Oral 2010
GS_SDS_Oralrevised03012010 (6/2/2010)

The Osler Institute


3. What tests do you want? Answer: U/S to see if ducts are dilated, implying you clipped CBD, or less likely, pancreatitis; HIDA scan to see if there is a leak. HIDA confirms a bile leak into subhepatic space and U/S shows a biloma in same area. 4. What next? Answer: Drain biloma percutaneously; ERCP to see if can delineate leak and stent across it. If ERCP shows cystic duct leak, will patient need to be reexplored? (Unlikely) If ERCP shows CBD laceration, what is your plan? Wait 2-3 weeks for inflammation to subside, then attempt to repair over T-tube or perform choledochoenterostomy.

Case 10
A 58-year-old male is referred to you by a family practitioner for a hemorrhoid that will not respond to usual conservative measures. You examine the patient and see a 2 cm fungating lesion in the anterior midline. 1. What do you do? Answer: Incisional biopsy under local; anoscopy/proctoscopy to R/O concurrent lesions; examine inguinal lymph nodes. 2. Pathology returns squamous cell carcinoma. What next? Answer: Complete staging of patient with pelvic CT to be sure any enlarged nodes are FNA'ed and included in radiation field. 3. Patient does not have enlarged pelvic or inguinal nodes. How do you treat? Answer: Patient receives two cycles of 5-FU and mitomycin as radiosensitizers, along with 3000-3800 cGy RT; biopsy area of prior tumor 4-6 weeks after completion RT. Post-treatment biopsy shows residual carcinoma. Can retreat with further RT and chemo. Use Cisplatinum instead of mitomycin (the preferred answer), or can offer APR. Latter is most appropriate in patient who already has significant anorectal complaints secondary to initial round of Chemoradiation.

Case 11
A 75-year-old female comes to your office with complaint of pencil-like stools and tenesmus. She has seen no blood in her stool and has no weight loss. Barium enema shows an "applecore" lesion of the midsigmoid colon, but the quality of the exam is poor due to inadequate prep. 1. What tests would you like? Answer: Liver function test, CEA, chest X-ray; colonoscopy to cecum to R/O synchronous cancers and biopsy sigmoid lesion; abdominal/pelvic CT to R/O liver mets and assess for adenopathy. The CT confirms the presence of the lesion, no adenopathy or liver mets is seen, but the left renal pelvis is
GS_SDS_Oralrevised03012010 (6/2/2010)

GS-SDS-Oral 2010

page 95

Surgery Review Course


slightly dilated. Colonoscopy reveals a carcinoma in the sigmoid and several other polyps, 2 cm or less, all of which are removed endoscopically and prove to be adenomas on path. Would you give this patient preoperative radiation and/or chemotherapy? Answer: NO. This is only being employed for the fixed portions of the rectum at this juncture. Describe your bowel prep. What other things might you want to do preoperatively? Answer: Anti-DVT precautions, stent left ureter since it may be involved in tumor mass. Upon entry into abdomen, you find a bulky sigmoid mass with adherent small bowel and invasion into the psoas muscle. There is no carcinomatosis and no liver mets. How do you remove this? Answer: Resect en bloc with small bowel and retroperitoneal structures. This leaves you with a significant loss of ureteral length on the left. How do you fix this? Answer: Can try bladder hitch, can cross ureter over to other side, but this risks infecting both kidneys, can bring out end of ureter as ureterostomy, can implant in left colon. Best alternative, if cannot do primary repair over stent with bladder hitch, may be to do nephrectomy if renal function normal preop and all studies confirm presence and function of contralateral kidney. Pathology shows this tumor to be invasive into the small bowel and the psoas muscle, with 10/23 nodes positive for cancer. What is the TNM stage of this cancer? Answer: T4N2M0, stage III, Aster-Collier C3 What additional treatment might you offer this patient? Answer: 5-FU/levamisole for one year, possibly radiation to the resected area What are this patient's chances of living 5 years without cancer recurrence? Answer: Less than 30% Discuss the pros and cons of subtotal colectomy as primary therapy for colon carcinomas.

2.

3. 4.

5.

6.

7. 8.

9.

10.

Case 12
A family practitioner sends you a 35-year-old female patient whom he has diagnosed as being hyperthyroid. There is no endocrinologist available. The patient is symptomatic with weight loss, tremulousness, heat intolerance, and palpitations.

page 96

GS-SDS-Oral 2010

GS_SDS_Oralrevised03012010 (6/2/2010)

The Osler Institute


1. What are the causes of hyperthyroidism? Answer: Grave's disease, toxic nodular goiter, hyperfunctioning adenoma, early stages of thyroiditis 2. What other lab studies would you like? Answer: LATS level if suspect Grave's, antithyroid antibody to R/O thyroiditis, thyroid scan to R/O hot nodule, U/S thyroid to R/O dominant mass. 3. What is the initial medical management? Answer: PTU or Tapazole - complication is agranulocytosis vs radioactive iodine - felt to be better option in older patients. 4. What are your indications for thyroidectomy in this patient? Answer: Wants to become pregnant (can't use PTU); compressive symptoms, e.g., dysphagia; dominant nodule failure or noncompliance with med therapy. The patient, who has no children, wants to become pregnant now. 5. How do you prep her for surgery? Answer: Continue antithyroid meds. Give beta-blocker if still tachycardic. Start iodine solution 10-14 days preop. Supposed to decrease vascularity of gland, but many surgeons omit. 6. What operation do you do if patients has toxic nodular goiter? Answer: Subtotal thyroidectomy. Describe how you do it.

Case 13
You do a needle-localized breast biopsy on a 42-year-old patient for grouped microcalcifications. The biopsy shows apocrine metaplasia with a 1 cm focus of lobular carcinoma in situ. 1. How would you counsel this patient? Answer: Emphasize bilateral breast risk. Estimated lifetime risk of developing invasive cancer 25-60%. Type of cancer developing is generally invasive ductal, not invasive lobular. Treatment is either close observation or bilateral simple mastectomy. No rationale for heretofore employed mastectomy and mirror-image biopsy. Radiation therapy is not given prophylactically in LCIS. The ability of Tamoxifen to prevent conversion into carcinoma is unknown. 2. What other factors would you consider that might make you encourage the patient to have a mastectomy? Answer: Strong family history of premenopausal breast cancer, evidence of ductal atypia elsewhere in specimen. Nulliparity alone probably not strong enough reason to encourage surgery. The patient desires bilateral mastectomy. 3. What reconstructive alternatives are available? Answer: TRAM vs implant after placement of tissue expander. Latissimus flap not used very often.

GS_SDS_Oralrevised03012010 (6/2/2010)

GS-SDS-Oral 2010

page 97

Surgery Review Course Case 14


A 48-year-old female comes to you with a history of infrequent bloody nipple discharge from her right breast. 1. What would you do first? Answer: Complete H & P with attention to risk factors for breast cancer. Feel for breast mass. Try to ascertain which areolar quadrant is involved by circumferentially milking breast. There is no breast mass and you cannot localize the duct. Get mammogram/ultrasound. 2. The mammogram and ultrasound are negative. What do you do next? Answer: Complete subareolar (major) duct excision, although intraductal papilloma is most likely cause. Risk of cancer with bloody nipple discharge in this age group may be approaching 10-15%. Variation may be that you can localize duct. If so, know how to operatively manage. Lacrimal probe in suspect duct or inject with methylene blue to mark arborization of duct. Ductogram and cytology on the discharge are marginally useful.

Case 15
You are surveying the abdomen of a 55-year-old female patient before commencing with a laparoscopic cholecystectomy when you notice a 4 cm mass on the right ovary. 1. What do you do? Answer: (No GYN available.) Solid mass in this age group is highly suspicious for cancer. Open patient and do ovarian cancer staging, bilateral salpingo-oophorectomy, (hysterectomy not needed), and omentectomy after doing peritoneal washings, scraping under-surface of diaphragms for cytology and biopsy of any apparent tumor implants on bowel or elsewhere.) Variation on theme: patient is premenopausal and of childbearing age. Do washings and scrapings first as biopsy can give false positive if it precedes washings. Incisional biopsy of mass, avoid spillage. Get frozen section, mandatory. If malignant, remove rest of ovary. Leave other ovary alone as some of these are sarcomas which do not require pelvic clearance, and, even if adenocarcinoma, some young women are treated without bilateral salpingo-oophorectomy.

Case 16
A 45 year-old man with a long history of aggressive peptic ulcer disease comes to see you complaining of worsening epigastric pain and diarrhea. On EGD he has multiple ulcers of the stomach and duodenum, extending into the third portion of the duodenum.

page 98

GS-SDS-Oral 2010

GS_SDS_Oralrevised03012010 (6/2/2010)

The Osler Institute


1. How would you work this up? Answer: Get serum gastrin off H2 blockers; gastric analysis to measure basal/stimulated gastric output; secretin test. Look for paradoxical abrupt increase in serum gastrin within minutes after secretin IV push. The blood work suggests gastrinoma. 2. What next? Answer: CT most important study to R/O mets to liver and to hopefully localize. Other helpful studies: arteriogram, portal venous sampling. No mets are seen on the CT, but all the localizing studies are negative. What next? Explore patient with especial attention to the gastrinoma triangle. If still can't find, intraop duodenoscopy and ultrasound as most occult gastrinomas are submucosal in duodenum. If still cannot find, consider parietal cell vagotomy to reduce ulcer diathesis, along with omeprazole or somatostatin. Variations: metastatic disease on CT. If only a few liver mets, can explore to try to debulk and do anti-ulcer operation; if numerous, medical management for ulcers and chemotherapy. Whipple procedure OK for sporadic form of Z-E (familial form seen in MEN syndrome is usually multifocal), where it is the only feasible alternative for complete disease excision.

Case 17
Right lower quadrant pain in a sexually active 24-year-old female. You do the usual workup and decide to take the patient to the operating room. You explore the patient through a transverse RLQ incision. The appendix is normal, but you encounter a markedly inflamed right ovary and tube with an 8 cm abscess. 1. What do you do? Answer: This is a tough question because, even within gynecological circles, there is no consensus regarding TOA management. Some argue for aspirating or incising the abscess and treating aggressively with 3rd generation cephalosporin and doxycycline in order to conserve the ovary. Others feel that not removing the abscessed ovary increases the risk of involving the contralateral ovary and can lead to bilateral oophorectomy. It's your call. Variations: unilateral/bilateral involvement, but only PID or hydrosalpinx without frank abscess. Okay to close and treat with previously mentioned antibiotics. Follow with serial ultrasound to R/O development TOA. 2. Bilateral abscesses Answer: Seems reasonable to attempt conservative management since both ovaries are already involved. Drain both sides and follow closely. Alternatively, if there is inflammatory involvement of the periuterine tissues, raising the possibility of pelvic cellulitis which can be a disaster akin to Fournier's gangrene, it is better to do the TAH and BSO, leaving the vaginal cuff open to drain the pelvis.
GS_SDS_Oralrevised03012010 (6/2/2010)

GS-SDS-Oral 2010

page 99

Surgery Review Course Case 18


A 50-year-old male rancher comes to see you because his wife noticed a new black skin lesion on the back of his thigh. Differential? Answer: Melanoma, benign nevus, pigmented wart, pigmented fibroma, basal cell carcinoma, squamous cell cancer. Last two less likely in this non-sun exposed area. Physical exam? Answer: Check for other suspicious lesions especially on ipsilateral foot (make sure thigh lesion not an in-transit met), and in BANS area. Check all lymph node basins. Note if primary lesion is ulcerated. How would you biopsy this? Answer: Can do excisional biopsy with 2-3 mm margin or punch biopsy into subcutaneous fat. The diagnosis returns superficial spreading melanoma, Breslow depth 2.0 mm, Clark's level IV. What is the difference between the Clark's and Breslow level? Answer: Breslow is simply the thickness of lesion measured with eyepiece micrometer. Clark's level is based on level of invasion into various levels of the skin architecture. What should you do now? Answer: Re-excise with 2-3 cm margin. Do you do superficial node dissection? Answer: No proof in any prospective trial that survival is improved, although if any group benefits, it is men with intermediate depth lesions (.78-4 mm Breslow) of extremities. If considering node dissection, do radionuclide immunoscintigraphy before re-excision. Want to know that lesion only drains to one lymph node basin. Also, R/O systemic mets with, at the least, chest X-ray and liver function tests. LDH most predictive of occult metastases. If lesion drains to right groin only and no distant mets, can do superficial node dissection. Know how to describe operation, its complications, or can do sentinel node biopsy. (See Morton's work from John Wayne Cancer Center in LA.)

1.

2.

3.

4.

5. 6.

Case 19
A 27-year-old professional football cornerback comes to your ER complaining of bloody emesis. He has been taking Motrin for various muscle complaints. 1. Initial therapy? Answer: Place on IVs, start fluids, draw T & C, check coags, place NG to confirm bleeding.

page 100

GS-SDS-Oral 2010

GS_SDS_Oralrevised03012010 (6/2/2010)

The Osler Institute


2. The NG aspirate is grossly bloody and continues to be so. What next? Answer: Perform endoscopy. A bleeding duodenal ulcer is seen with a visible vessel in the base, which you inject with epinephrine. 3. The bleeding appears to stop. What next? Answer: Admit for close observation and start on H2 blockers/Carafate. 4. The patient continues to require a unit of blood daily to keep his H/H at 10/30, and he then has a four-unit bleed on hospital day #6. What next?
Answer: Can re-endoscope because patient has not required 6-unit transfusion in 24-hour period. More than likely should explore in OR since this is a re-bleed situation.

5. How do you perform the operation? Answer: Open duodenum with pyloroplasty incision. Four quadrant "U" sutures to ligate gastroduodenal artery and longitudinal pancreaticoduodenal vessel. The most I would do is parietal cell vagotomy and re-close pylorus. Could argue no anti-ulcer operation is needed since cause of bleed may well have been NSAIDs. Variations: older patient with high surgical risk-truncal vagotomy and pyloroplasty after oversewing ulcer. Younger patient with long history of recalcitrant PUD-consider vagotomy and antrectomy.

Case 20
A 90-year-old nursing home patient brought to your ER with distended abdomen and no bowel movement for three days. After initial resuscitation, you are called to assess the patient. Differential? Answer: Cancer, diverticulitis, volvulus, pseudo obstruction. Obstructive series show dilated colon and multiple air-fluid levels in the small bowel. What next? Answer: Sigmoidoscopy as it can diagnose diverticulitis or a distal cancer. Can be therapeutic for sigmoid volvulus. Sigmoidoscopy is non-diagnostic. What next? Answer: Gastrografin enema. (Don't want to use barium for fear of perforation.) Enema reveals an obstructing lesion of the midsigmoid. Now what? Answer: Would like to be able to bowel prep patient and clear rest of colon before surgery, but more likely, serial obstructive series will show increasing cecal distension.

1.

2.

3.

4.

GS_SDS_Oralrevised03012010 (6/2/2010)

GS-SDS-Oral 2010

page 101

Surgery Review Course


5. The next morning, after rehydration and NG suction, the patient's cecum now measures 10 cm and her abdomen is more tender. What do you want to do? Answer: Take patient to OR. Have several choices for obstructing left colon lesion: (1) excise sigmoid cancer and divert with end colostomy and mucous fistula or Hartman's pouch; (2) sigmoidectomy, on-table lavage, and primary anastomosis; and (3) subtotal colectomy, irrigate rectum, ileorectal anastomosis. Variants: obviously diverticulitis, sigmoid volvulus. With pseudo-obstruction, watch cecal diameter. If less than 10 cm, use rectal tube, NG, serial X-rays. If 10 cm, but no peritoneal signs, decompress colonoscopically and observe. Over 10 cm but unstable and/or peritoneal signs, cecostomy if no ischemic segment vs segmental resection for perf/ischemia with ileostomy and mucous fistula. Case 21 A 75-year-old female who underwent sigmoidectomy for diverticulitis one year ago has noted a 20-pound weight loss since OR, occasional night sweats and recurrent urinary tract infections. What are the possible etiologies for these complaints? Answer: Subacute infectious symptoms mean need to consider persistent abscess from anastomotic leak or retained foreign body (like lap sponge). Path diagnosis incorrect and patient had cancer removed and now has recurrence, enterovesical fistula. Workup? Answer: Plain films of abdomen to R/O foreign body; urine culture: polymicrobial infection implies fistula; colonoscopy to R/O cancer. The urinalysis shows 100,000 colonies of four different enteric bacteria. What next? Answer: CT of abdomen/pelvis to R/O intervening abscess and assess anatomy. Cystoscope: usually cannot see fistula, but see heaped-up mucosa in area. Barium enema and X-ray urine after to see if barium in bladder. IVP to delineate fistula relationship to bladder/ureters. The barium enema demonstrates a colovesical fistula and the CT shows no intervening abscess. Your operative approach? Answer: Stent left ureter preop. Take fistula down from bladder and oversew, drain bladder with Foley for 7-10 days. Sigmoid resection in area of fistula and perform primary anastomosis.

1.

2.

3.

4.

page 102

GS-SDS-Oral 2010

GS_SDS_Oralrevised03012010 (6/2/2010)

The Osler Institute Case 22


A patient who underwent vagotomy and antrectomy 6 months ago comes to see you complaining of severe abdominal cramping 30 minutes after most meals, often forcing her to lay down for an hour. 1. What is happening?
Answer: (Patient has dumping syndrome.)

2. What causes dumping? Answer: Not totally understood. Loss of stomach reservoir function caused too prompt delivery of high solute load to small bowel causing outpouring of vasoactive enteric hormones and also leading to fluid sequestration in the bowel. 3. What is the difference between early and late dumping? Answer: Early dumping is characterized by the symptoms of GI hypermotility (cramping, diarrhea, nausea, vomiting) and cardiovascular instability (sweating, pallor, fainting). Late dumping occurs about 2 hours after meal and the symptoms are those of hypoglycemia. 4. How do you treat dumping? Answer: Dietary modification including low carbohydrate, high fat/protein meals, limitation of dairy products, limited fluid with meals. Some are helped with anticholinergic drugs or by supplemental protein. 5. If dietary modification does not suffice, what are your operative alternatives? Answer: If previous pyloroplasty, reverse pyloroplasty; convert BI to BII (or vice versa); antiperistaltic; 10 cm small bowel segment interposed at gastric outlet.

Case 23
Previously healthy 15-year-old female comes to your office complaining of vague RLQ pain for 3 days. She has been having night sweats. Her LMP was 2 weeks ago, and about two weeks ago she had a gastroenteritis that got better by itself after 48 hours. 1. Differential? Answer: Appendix; ovarian pathology; less likely, but with night sweats, have to think neoplasm, especially lymphoma. 2. What labs would you like? Answer: CBC, U/A, obstructive series. WBC is 16,000 with left shift. U/A shows 10-15 RBCs, but only 2-3 WBCs. Abdominal series shows air dilated small bowel loops. 3. What do you suspect? Answer: Intra-abdominal abscess given GI complaints. Ileus on KUB; elevated WBC.

GS_SDS_Oralrevised03012010 (6/2/2010)

GS-SDS-Oral 2010

page 103

Surgery Review Course


4. What test do you want next? Answer: Abdominal CT/US to look for abscess. 5. CT shows RLQ abscess. How treat? Answer: Percutaneously drain. Place on antibiotics. Know which ones you want to use and for how long. 6. What is the probable source of the abscess, and how might you prove this? Answer: Ruptured appendix; can try to see if appendix fills on barium enema (not necessary). 7. What are your indications for doing an interval appendectomy and when would you perform it if you decide it is necessary?

Case 24
A 56-year-old man who underwent left hemicolectomy for a stage II colon cancer three years ago now returns with a solitary liver metastasis to the right lobe. He is referred to you for possible metastasectomy. 1. What percentage of patients who undergo hepatic metastasectomy for colon cancer will survive 5 years? Answer: About 25% 2. What are some of the factors that predict for a better outcome after metastasectomy? Answer: Female, primary tumor stage II or less, less than three liver mets, interval between removal of primary, and appearance of liver mets over one year. 3. If you feel this gentleman is a candidate, what further workup do you want? Answer: Need to prove there is no extrahepatic disease because there is no benefit to metastasectomy in those with disease outside liver. CEA level, abdominal/ chest CT, bone scan if alkaline phosphatase elevated, colonoscopy to check anastomosis, monoclonal antibody scan (OncoScint, or CEAlabeled) to image occult mets.) If no evidence of extrahepatic disease, describe how you do surgery. Key points: do intraoperative ultrasound of liver to R/O unsuspected mets; scrupulously search for extrahepatic mets, especially hepatic artery nodes. If these are positive for tumor, do not resect. No benefit of anatomic resection over simple nonanatomic metastasectomy as long as margins are clear; know landmarks of various major resections: right lobectomy, trisegmentectomy and left lobectomy.

Case 25
40-year-old male who underwent "some surgery" on his stomach for ulcers 5 years ago and now is starting to have pains similar to his preop ones. He reports no vomiting, weight loss, or early satiety. Food intake initially decreases pain, but within hours it returns.
page 104 GS-SDS-Oral 2010
GS_SDS_Oralrevised03012010 (6/2/2010)

The Osler Institute


1. What do you need to do first? Answer: The type of operation he underwent is critical. Either obtain the operative report or do an UGI. 2. The patient had a vagotomy and antrectomy with Billroth II reconstruction. What could be the causes of his pain? Answer: Incomplete vagotomy, retained antrum, inadequate antrectomy, marginal ulcer/outlet obstruction, missed gastrinoma, hypercalcemia (possibly part of MEN), druginduced (eg, NSAIDs), bile reflux gastritis, Helicobacter. 3. What blood work would be helpful? Answer: Serum gastrin to R/O gastrinoma and retained antrum; serum calcium. 4. What information might you want from the path reports from his initial surgery? Answer: Check to see if vagi were confirmed by path; check to see if distal gastric margins showed goblet cells of duodenum. 5. Gastrin and calcium levels are non-elevated. EGD shows no marginal ulcer; Helicobacter assay is negative; a random gastric pH is 3. What next? Answer: Test for incomplete vagotomy with sham-feeding test. Do gastric acid output analysis under three conditions: (1) fasting, (2) chewing on food, and (3) after IVP Pentagastrin. His basal acid output is greater than 4 mmol/hr and stimulated output is 20% of peak Pentagastrin output. 6. Diagnostic of incomplete vagotomy. What do you do next? Answer: Might want to check serum gastrin after feeding test. If increases 2-3X over baseline with po, could indicate concomitant antral cell hyperplasia. No matter what, try aggressive medical management first, augmented by dietary modification. 7. Despite Omeprazole and PRN antacids, patient is still complaining of almost constant pain. What next? Answer: If indication of insufficient antrectomy and vagotomy, re-explore via abdomen and revise gastric pouch and complete vagotomy. If insufficient vagotomy alone, can attempt thoracoscopic truncal vagotomy. Treatment: Suppressive Antibiotics Thiazide Diuretic

GS_SDS_Oralrevised03012010 (6/2/2010)

GS-SDS-Oral 2010

page 105

Surgery Review Course Case 26


A 78-year-old white female complains of intermittent, crampy, generalized abdominal pain with vomiting for three days. She has been unable to eat, and for the last day, has been unable to take liquids. She has had intermittent vomiting, worse over the last day, of greenish-brown foul smelling liquid in large amounts. Her last bowel movement was yesterday, and she cannot remember passing flatus. Her past medical history is unremarkable, and she has never had surgery. On physical examination, she appears moderately dehydrated with normal vital signs except for a pulse of 118. Her abdomen is moderately distended with very few high-pitched bowel sounds. She is tender to deep palpation generally, with no involuntary guarding, percussion or rebound tenderness. No masses are present. Rectal examination is negative, with an empty rectal vault.

Case 27
An eight-year-old boy presents to the emergency room with an eight hour history of bloody diarrhea, with four to six maroon to reddish liquid moderately sized bowel movements per hour. He has had abdominal cramping, mostly both lower quadrants after onset of the diarrhea. His past medical history is unremarkable, with no prior surgical history. On physical examination, he is pale, anxious, and confused, with tachycardia and hypotension. Examination of the abdomen is essentially normal, except for grossly bloody stool in an otherwise normal rectum.

Case 28
You are asked to see a 72-year-old white male in the emergency room who complains that he has a piece of meat stuck in his esophagus since his evening meal three hours ago. He is unable to swallow anything, including his own saliva. His past medical history includes frequent episodes of heartburn over several years, for which he often takes bicarbonate of soda with relief. On examination he is slightly anxious, with normal vital signs, spitting his saliva in a cup. Examination is otherwise unremarkable.

page 106

GS-SDS-Oral 2010

GS_SDS_Oralrevised03012010 (6/2/2010)

The Osler Institute Case 29


You are asked to see a 32-year-old white male who was brought to the emergency room by ambulance, who collapsed at work with excruciating upper abdominal pain of sudden onset. His history includes heavy smoking and alcohol use, as well as frequent use of over-the-counter antacids for heartburn, especially over the last several months. On physical examination he is anxious, tachycardic, hypertensive, and complains of severe upper abdominal pain. His abdomen is silent, rigid, and exquisitely tender to palpation, especially in both upper quadrants. Examination is otherwise unremarkable.

Case 30
You are asked to see a 52-year-old white male in the emergency room who complains of severe upper abdominal pain and shortness of breath. The pain began approximately 48hours ago after a bout of heavy drinking, and has escalated in intensity over the last few hours. The patient relates to you that he believes the pain is due to some rot gut liquor he obtained which made him vomit repeatedly. The abdominal pain is sharp, egigastric, constant, severe, and radiates to the back and left chest. He is anorexic, and complains that he feels short of breath. Past medical history includes chronic alcoholism with multiple visits to the Emergency Room for trauma resulting from beatings on the street, and a two week hospitalization for pneumonia last winter. No other significant past medical history is obtainable. On physical exam, the patient is tachypneic, febrile, tachycardic, and slightly hypotensive. He appears flushed, anxious, and cannot lie flat due to dyspnea. There are diminished breath sounds in the left base, with rales in the left mid lung field. The abdomen is slightly distended, nearly silent, and moderately tender to direct palpation in all quadrants, more so in the epigastric area. There is no involuntary guarding, percussion tenderness or rebound tenderness. Rectal examination is negative except for trace Hemoccult positive.

GS_SDS_Oralrevised03012010 (6/2/2010)

GS-SDS-Oral 2010

page 107

Surgery Review Course Case 31


You are called to see a 65-year-old male who was admitted to your hospital by a general practitioner for hematosis and weakness 24 hours ago. He has been treated for peptic ulcer twice over the last six or seven years with H-2 blockers successfully, but relates that one week prior to admission he began having particularly severe epigastric burning which diminished incompletely with eating or medication. On the day of admission, he reported having tarry stools, feeling nauseated, and vomiting coffee-ground material twice prior to coming to the hospital. On admission, he was tachycardic, hypertensive, pale and anxious, with mild deep epigastric tenderness and grossly melanotic stool. His admission hemoglobin was 6.2 gm/dl. He was resuscitated with crystalloid IV fluid and four units of packed RBCs to a hemoglobin of 10.1 gm/dl. After initially feeling better, with no further vomiting or melena, he became nauseated, and had three small melanotic foul smelling stools this evening. His nurse was instructed to consult you when she notified his general practitioner of these events by telephone.

page 108

GS-SDS-Oral 2010

GS_SDS_Oralrevised03012010 (6/2/2010)

Potrebbero piacerti anche